Вы находитесь на странице: 1из 113

STEP

1.Which of the following is not correct about dermatologic manifestations of congenital generalized
lipodystrophy?

A. Acanthosis nigricans

B. Xanthomas

C. Hyperkeratotic epidermal papillomatosis

D. Hypohidrosis

E. Coarse skin of the upper body

2.Which disease is not associated with epidermolysis bullosa acquistia ?

A. Hyperthyroidism

B. Tuberculosis

C. Bronchial carcinoma

D. Diabetes mellitus

E. Amyloidosis

3.One of the following drug does not cause Pityriasis Rosea - like drug eruptions:

A. Gold.

B. Antimalarials.

C. Beta- blockers

D. Metronidazole.

E. ACE inhibitors.

4.One of the following drug does not cause alopecia :

A. Lithium

B. Acitretin.

C. Colchicine.

D. Allopurinol.

E. Proylthiouracil.
5.Which of the following is not correct about Fabry's disease?

A. X-linked recessive disorder due to deficiency of alpha–galactosidase A

B. Multiple angiokeratomas

C. Hyperhidrosis

D. Renal and coronary insufficiency

E. Pain and paresthesia of the extremities

6.Which disease is least seen with skin ulceration?

A. Pyoderma gangrenosum.

B. Lymphomatiod papulosis.

C. Necrobiosis lipiodica.

D. Erythema induratrum.

E. Erythema nodosum

7.Which of the following is not a structural defect of the hair shaft with increased fragility?

A. Monilethrix.

B. Pili torti.

C. Trichorrhexis nodosa.

D. Pili annulati.

E. Trichothiodystrophy.

8.Which statement is not correct about invasive squamous cell carcinoma?

A. The common clinical presentation is an erythematous keratotic papule or nodule.

B. Tumors may have an exophytic and an endophytic component.

C. The metastatic potential is high in the solar-induced form.

D. High-risk sites for metastasis include in the lip and ear.

E. Lesions arise in scars and sites of inflammation are the most prone to metastasize
9.One of the following disease is not associated with anti-RO antibody:poo

A. Sjogren's syndrome.

B. C2- deficient lupus.

C. Neonatal lupus.

D. Waldenstrom hyperglobulinemic purpura.

E. Lupus profundus

10.Which statement is not true about Etanercept ?

A. It is indicated for psoriatic arthritis.

B. It is an inhibitor of T-cell activation

C. The dose is 50 mg /kg/wk SC.

D. Can be given to children >4 years of age.

E. One of the side effects is worsening of chronic heart failure.

11.Which statement is not correct about Becker's melanosis (nevus) ?

A. The lesions commonly have a unilateral distribution.

B. The most common configuration is block-like.

C. Malignant transformation has been reported.

D. The abnormalities that associated with Becker's nevus include hyperplasia of the ipsilateral breast
and ipsilateral arm shortening.

E. The hyperpigmented component may be treated with Q-switched ruby and frequency-doubled Nd-
YAG laser.

12.Which of the following is not a marker for infantile hemangioma ?

A. GLUT-1

B. Merosin

C. Human placental lactogen

D. Lewis Y antigen

E. FcVRII
13.Proximal splinter haemorrhages are not seen in :

A. Onychomatricoma.

B. Trichinosis.

C. Bacterial endocarditis.

D. Vasculitis.

E. HIV infection.

14.The classical localization of segmental vitiligo in children is:

A. Perineal and perianal.

B. Face.

C. Back.

D. Abdomen.

E. Upper limbs.

15.Which of the following skin condition is the least complicated by Kaposi's varicelliform eruption?

A. Atopic eczema.

B. Darier's disease.

C. Lichen planus.

D. Burn

E. Mycosis fungoides (Sezary).

16.Which of the following is not correct about Klipple-Trenauny syndrome ?

A. Port wine malformation.

B. Deep venous malformation.

C. Soft tissue hypotrophy.

D. Claudication.

E. Superficial varicosity.
17.Which statement is not correct about type 2 leprosy reactional state ?

A. Systemic small vessel vasculitis.

B. Severe joint swelling and pain.

C. Iridocyclitis.

D. Orchitis.

E. The treatment of choice is prednisone

18.Which statement is not true about Granuloma faciale ?

A. Common in males.

B. No systemic manifestations.

C. Soft brown-red nodules or plaque on face with prominent follicular orifices.

D. Lesions may ulcerate

E. Grenz zone is present in histopathology.

19.Hydroxyurea works by inhibiting which enzyme?

A. Ribonucleotidereductase

B. DNA gyrase

C. Dihydrofolatereductase

D. Inosine monophosphate dehydrogenase

E. Thymidine kinase

20.Which of the following has been reported as cutaneous components of phakomatosis


pigmentovascularis (typeII)?

A. Mongolian spots

B. Hypotrichosis

C. Lipohypoplasia

D. Nevus Sebaceus

E. Hypoplastic nails
21.The common age of giant cell histiocytoma is :

A. 0 – 2 y

B. 3 – 7 y

C. Any age

D. adult

E. elderly

22.An 84 old woman with a 6 months history of asymptomatic rapidly increasing brown macules in her
legs developed 2 months later a red nodule with central ulceration on her right shoulder. Blood tests
were normal and skin biobsy showed atrophic epidermis, lymphocytic infiltration at the dermo-
epidermal junction with epidermotropism and anaplastic cells in the basal layer. Immunohistochemistry
was positive for CD3+. CD8+, CD30+ , CD4 - .what is the diagnostic?

A. MF

B. Sezary synd

C. CD30+ LYMPHOPROLIFERATION

D. Primary aggressive CD8+ T cell lymphoma

E. pagetoid T cell lymphoma


23.An 84 old woman with a 6 months history of asymptomatic rapidly increasing brown macules in her
legs developed 2 months later a red nodule with central ulceration on her right shoulder. Blood tests
were normal and skin biobsy showed atrophic epidermis, lymphocytic infiltration at the dermo-
epidermal junction with epidermotropism and anaplastic cells in the basal layer. Immunohistochemistry
was positive for CD3+. CD8+, CD30+ , CD4 - .what is the diagnostic?

A. MF

B. Sezary synd

C. CD30+ LYMPHOPROLIFERATION

D. Primary aggressive CD8+ T cell lymphoma

E. pagetoid T cell lymphoma

24.Bacillary angiomatosis though a cutaneous disease may involve internal organs. which is most often
involved ?

A. lungs

B. brain

C. liver

D. kidneys

E. peripheral nerves

25. peripheral nerve changes in leprosy are :

A. symmetrical nerve enlargement

B. nerves trunk palsies without overt clinical manifestation ( silent palsy )

C. stocking – gloves pattern of loss of sensation due to proliferation of c fibres.

D. Anhydrosis of palms and soles due to parasympathetic nerve involvement

E. Cranial nerves are never involved


26.UVR interacts with the skin in the following manner :

A. 50% is diffusely reflected

B. Radiation above 300nm are removed within the epidermis

C. DNA is the most important chromophore

D. Collagen bundles patialy absorb UV

E. Cell photoproducts are completely repaired by vit E

27.dapsone topical gel has proven to be effective in inflammatory acne because it:

A. exerts an antibacterial effect

B. reduces the oiliness of the face

C. has a strong comedonlytic effect

D. inhibits leukocytes traffic

E. changes the PH of the skin

28. Periorbital is the most common site involved in :

A. Necrobiotic xanthogranuloma

B. Papular xanthogranuloma

C. Eiosinophilic granuloma

D. Chediak – higashi syndrome

E. Juvenile xanthogranuloma

29. The associated condition of acrocyanosis is :

A. Malignancy

B. Erythromalcia

C. Cryoprotiens

D. Trauma

E. Blood dyscrasia
30.Which of the following is not true about PLE ?

A. IT is the least common photodermatosis

B. It occurs within hours of sun exposure

C. Most severe in spring

D. Action spectra : UVB. UVA

E. Antimalarial can be used in treatment

31. Pityriasis Rosea – like eruption may be caused by :

A. Lithium

B. Cu

C. Gold

D. Zn

E. Irons salts

32.One of the following is not correct about chediak - Higashi syndrome:

A. AR with LYST gene mutation

B. Giant intracytoplasmic granules

C. Most common skin infection are superficial pyoderma

D. About 15% of patients suffer from hemophagocytic syndrome which leads to death by 10 year of age

E. Photophobia and nystagmus , but normal visual acuity

33. Cobalt toxicity may leads to :

A. Coagulopathy

B. Diffuse nodular fibrosis

C. Burns

D. Lichenoid drug eruption

E. Hepatic failure
34. Which of the following is false about chondrodermatitis nodularis helicis :

A. Age below 40 y

B. Unilateral tender nodule

C. Actinic damage is a predisposing factor

D. Mostly on upper helical rim

E. Acanthosis, parakeratosis and hypergranulosis.

35.Needle- shaped clefts in lipocytes, Histologically favors the dg of :

A. Pancreatic panniculitis

B. traumatic panniculitis

C. nodular vasculitis

D. poststeroidal panniculitis

E. lupus panniculitis

36.The most common inherited disorder associated with BCC is :

A. Bazex syndrome

B. Li - fraumeni syndrome

C. Nevus sebaceous

D. Gorlin syndrome

E. Keratoacanthoma.

37.which of the following retinoids is used to treat Kaposi sarcoma :

A. tazarotene.

B. Adapalene

C. Bexarotine.

D. Acitretin

E. Alitretinoin
38. peripheral nerve changes in leprosy are :

a. symmetrical nerve enlargement

A. nerves trunk palsies without overt clinical manifestation ( silent palsy )

B. stocking – gloves pattern of loss of sensation due to proliferation of c fibres.

F. Anhydrosis of palms and soles due to parasympathetic nerve involvement

G. Cranial nerves are never involved

39. which of the following is true about venous leg ulcer ?

A. occurs at pressure sites

B. A punched out ulcer

C. Leg and ankle odema may be seen

D. Shiny atrophic skin

E. Peripheral neuropathy

40.In reactive perforating collagenosis , which of the following is false:

A. It begins during childhold

B. Keratotic papule develop after superficial trauma

C. Koebernisation is commonly seen

D. Lower extremities are common sites involved

E. It resolves spontaneously in 6 – 8 weeks

41.The quantity of ointment required to treat 3 – 5 year child leg and foot is :

A. 1.5 FTUs

B. 2 FTUs

C. 3 FTUs

D. 4 FTUs

E. 5 FTUs
42. Which of the following laser, the target chromophor is vascular :

A. Argon

B. Erb- YAG

C. Pulsed dye

D. Alexandrite

E. Co2

43.Which of the following is false regarding risk factors of arterial ulcers ?

A. Peripheral neuropathy

B. Male gender

C. Hyperlipidemia

D. Hyperhomocysteinuria

E. Smoking

44.Which of the following is not side effects of systemic steroids?

A. Hypokalemic alkalosis

B. Hypocalcemia

C. Lymphocytosis

D. Pseudotumor cerebri

E. Exophthalmosis

45.Which of the following lasers is not used for treatment of nevi ?

A. Q switched ND :

B. LONG pulsed ruby

C. A

D. KTP

E. DIODE
46.Total skin electron beam therapy is used to treat selective patient of :

A. Kaposi sarcoma

B. Merkel cell Carcinoma

C. Malignant melanoma

D. Cutaneous T -cell lymphoma

E. SCC

47.Waldenstrom hypergamma globulinemic purpura is most frequently associated with:

A. Rheumatoid arthritis.

B. Lupus erythematosus.

C. Lymphoma.

D. Multiple myeloma.

E. Sjogren s syndrome

48. One of the following variants of acne is typically not comedonal:

A. Chloracne.

B. Acne mechanica.

C. Neonatal acne.

D. Infantile acne.

E. Acne vulgaris.

49.Concerning pigmented purpuric dermatoses, women are more affected than men in:

A. Schamberg s disease.

B. Purpura annularis telangiectoides of Majocchi.

C. Eczematid like purpura of Dukas and Kapetanakis.

D. Pigmented purpuric lichenoid dermatitis of Gougerot and Blum.

E. Lichen aureus.
50.One of the following is not a feature of sweat secreted by apocrine glands:

A. sterile

B. odorless

C. dilute

D. PH 5 – 6.5

E. Continuous secretion

51.Elderly patient with hyperpigmentd patch of lentigo maligna on the face, after simple excision he
require radiotherapy in a dose range :

A. 10 – 15 Gy in 5 fractions

B. 12 – 16 Gy adjuvant therapy

C. 40 – 50 Gy in 10 - 20 fractions

D. > 60 Gy in 25 fractions

E. > 100 Gy in 40 fractions

52.The longest peak effect of interferons is seen in :

A. Interferon alpha2a

B. Interferon a N3

C. Interferon Alphacon-1

D. Interferon 1b

E. Interferon Beta1b

53.CREST synd is a manifestation of complement disorder associated with deficiency of:

A. C1q

B. C2

C. C3

D. C5

E. C7
54.The most common cause of paraneoplastic pemphigus in children is:

A. Sarcoma.

B. Castleman s disease.

C. Non Hodgkin slymphoma.

D. Chronic lymphocytic leukemia.

E. Malignant and benign thymomas.

55.Which of the following signs/symptoms would support a diagnosis of SLE in a patient?

A. Annular eruption on upper chest and forearms

B. Presence of a false-positive VDRL

C. Presence of anti-Ro (SSA) antibodies

D. 0.25 grams/day of proteinuria

E. Elevated platelet count

56.Which of the following is most likely to cause a decreased therapeutic response to


hydroxychloroquine?

A. A deficiency of thiopurine methyltransferase (TPMT)

B. Co-administration of erythromycin

C. Advanced liver disease

D. Smoking

E. Co-administration of antacids or iron tablets

57.A 24 y/o Caucasian female presents with a history of juicy, erythematous papules in a sun exposed
areas. She says that the rash has waxed and waned and that it is very pruritic. Which of the following is
true of this condition?

A. This eruption is frequently caused by G-CSF.

B. 10 - 20% of patients with this disorder can have a positive ANA

C. Actinic cheilitis is a frequent finding

D. Hardening is never seen

E. Lesions typically appear 15-30 minutes after exposure to sunlight


58.Which of the following is not a finding that supports the diagnosis of neurofibromatosis?

A. First-degree relative with neurofibromatosis

B. Axillary freckling

C. 3 Lisch nodules

D. Optic glioma

E. 4 café-au-lait macules

59.While undergoing treatment for borderline leprosy, a patient develops swelling and ulceration of his
leprosy lesions. Which action would be undertaken?

A. Start on oral steroids

B. Start on thalidomide

C. Increase dose of current leprosy meds

D. Half dose of current leprosy meds

E. Administer fresh frozen plasma and vitamin K

60.Pyogenic granuloma : which of the following statements is false?

A. Commonly occurs on the gingival, lips, or fingers.

B. Usually develops at the site of preexisting injury.

C. May appear within a preexisting nevus flammeus.

D. With multiple lesions has a predilection for the interscapular region.

E. Drug – indeced pyogenic granuloma tendo to persist after withdrawal of the causative drug.

61.What is the major function of urocanic acid?

A. Bacteriocidal acid produced by stratum corneum

B. Primarily a UVB filter

C. Primarily a UVA filter

D. Helps degrade free fatty acids

E. Aids in protecting the skin from dermatophytes


62.Which of the following is NOT TRUE concerning multicentric reticulohistiocytosis?

A. Characterized histologically by histiocytes and giant cells with ground-glass cytoplasm

B. Frequently associated with mutilating arthritis

C. Represents a paraneoplastic syndrome approximately 25% of the time

D. Systemic chemotherapy is required in the majority of cases

E. Periungual papules in a coral bead arrangement are characteristic

63.Which of the following statement is not true about sarcoidosis ?

A. Erythema nodosum is the most common non specific cutaneous finding.

B. Lupus pernio may resemble rhinophyma.

C. Stage III systemic sarcoidosis includes pulmonary infiltrate and adenopathy.

D. Systemic steroid is beneficial in cutaneous sarcoidosis.

E. Granulomatous uvietis can occur

64.Renal involvement in Henoch – Schonlein purpura:

A. Is second, after skin, in frequency of appearance.

B. Tends to occur early in the course.

C. Never progresses to renal failure if the only presenting sign is microscopic hematuria.

D. Is unlikely to develop later is renal manifestations are absent in the first episode of HSP.

E. Is always associated with linear deposit of IgA in kidney biopsy specimens


65.Each of the following statements is not true of inflammatory bowel disease (IBD) and pyoderma
gangrenosum ?

A. IBD occurs in about one third of patients with ulcerative pyoderma gangrenosum.

B. Ulcerative colitis and Crohn's disease occur with equal frequency in patients with ulcerative pyoderma
gangrenosum.

C. IBD may precede, follow, or occur simultaneously with pyoderma gangrenosum.

D. Total protocolectomy is the treatment of choice.

E. Pustular pyoderma gangrenosum is strongly associated with IBD.

66.Staphylococcal and streptococcal toxic shock syndromes differ in that:

A. Streptococcal toxic shock is never fatal.

B. Only staphylococcal toxic shock is caused by the release of exotoxins.

C. Streptococcal toxic shock is more commonly associated with invasive infections.

D. Only staphylococcal toxic shock can cause hypotension.

E. Staphylococcal toxic shock is a far more serious condition.

67.Which of the following statements is false about telangiectases ?

A. An increased number of estrogen and progesterone receptors are present in unilateral nevoid
telangiectasia

B. Generalized essential telangiectasia is more common in women

C. Hereditary hemorrhagic telangiectasia is transmitted as an autosomal dominant condition

D. An increased number of estrogen and progesterone receptors are present in lesions of generalized
essential telangiectasia

E. There is a benign form of hereditary telangiectasia with no bleeding diathesis


68.Merkel cell carcinoma is associated with:

A. Frequent local relapses

B. Low metastatic potential

C. Excellent response to wide local excision in all clinical stages

D. Distant organ spread preceding regional lymph node spread

E. Lack of in vitro sensitivity to cisplatin

69. One of the following clinical manifestations is uncommon in Behçet's disease:

A. Hemorrhagic blisters

B. Palpable purpura

C. Papulopustular lesions

D. "bone white" scars

E. Erythema nodosum–like eruptions

70. Which of the following characterizes the usual course of histiocytic necrotizing lymphangitis
(Kikuchi's disease)?

A. Response to antibiotics

B. Response to chemotherapy

C. Response to radiation therapy

D. Response to topical corticosteroids

E. Spontaneous regression

71. The minocycline hypersensitivity syndrome consists of fever, eosinophilia, lymphadenopathy,


maculopapular exanthem, hepatitis, and one of the following:

A. elevated creatinine kinase level

B. increased pulmonary capillary pressure

C. elevated hepatitis B surface antigen levels

D. lymphocytosis

E. hemolytic anemia
72. Which of the following is a late manifestation of congenital syphilis ?

A. Papulosquamous exanthem

B. Hepatomegaly

C. Saddle nose

D. Lymphadenopathy

E. Periostitis

73. One of the following statements about p53 protein is false:

A. The wild-type protein has a half-life of 20 to 30 minutes.

B. Accumulation of p53 protein retards the formation of apoptosis.

C. p53 messenger RNA increases 10- to 20-fold late in the G1 phase.

D. The gene for p53 protein is on chromosome 17.

E. Immunohistochemical procedures detect mutated p53 gene product.

74. One of the following nail changes is not associated with lichen striatus:

A. Longitudinal ridging

B. Longitudinal splitting

C. Spooning defect

D. Onychomycosis

E. Striate leukonychia

75. Safe and effective treatment of Sweet's syndrome include one of the following:

A. Interferon

B. Cimetidine

C. 13-cis-retinoic acid

D. Indomethacin

E. Plaquenil
76. Green hair:

A. May result from incorrect shampooing

B. Is seen in long-distance runners

C. May result from tight football helmets

D. May be seen in those who swim regularly

E. Is the result of algae bloom

77. Which of the following statements about solar urticaria is correct?

A. Solar urticaria is a constant feature of lupus erythematosus.

B. Solar urticaria may be caused by UVB, UVA, or visible radiation.

C. Solar urticaria does not respond to treatment with oral antihistamines.

D. Most cases are unrelated to IgE antibodies.

E. Usually subsides within a year or two without treatment.

78. Piezogenic papules are:

A. Always painful

B. Herniations of muscle into the dermis

C. Prominent on sitting

D. Occur in long-distance runners

E. Hereditary

79. In busche – Loewenstein tumour one statement is true:

A. Tends to appear in circumcised men

B. Is said to represent 80% to 95% of all penile cancers..

C. Does not seem to show a preference for either circumcised or uncircumcised men.

D. Is said to represent 5% to 24% of all penile cancers.

E. Radiotherapy is the trestment of choice.


80.Eosinophilic pustular folliculitis has been reported to respond to which of the following therapies?

A. Isotretinoin.

B. UVA phototherapy.

C. Hydroxyzine.

D. Ibuprofen.

E. Hydroxychloroquine.

81.A feature of congenital erythropoietic porphyria that distinguishes it from porphyria cutanea tarda is:

A. Splenomegaly

B. Decreased ferrochelatase levels

C. Increased excretion of porphobilinogen and d- aminolevulinic acid

D. Elevated levels of hepatic transaminases

E. Potential for scarring after resolution of lesions

82.Pentostatin (Nipent), a purine analogue is an ideal drug for:

A. Leprosy

B. Low-grade T-cell malignancies

C. Urticaria pigmentosa

D. Atopic eczema

E. Psoriasis

83.One of the following is not a feature of multiple endocrine neoplasia III (Froboese's syndrome):

A. Medullary thyroid carcinoma

B. Pheochromocytoma

C. Pancreatic islet tumors‫ة‬

D. Mucosal neuromas

E. Marfanoid features
84.In an HIV-infected person, which of the following cutaneous disorders is associated with the highest
CD4 cell count?

A. Herpes zoster

B. Acquired ichthyosis

C. Molluscum contagiosum

D. Eosinophilic folliculitis

E. Kaposi's sarcoma

85.One of the following viruses has not been postulated in the origin of Kaposi's sarcoma:

A. Cytomegalovirus

B. Papillomavirus

C. Epstein-Barr virus

D. Coxsackie virus

E. Gamma herpes viridae

86.Periodic acid–Schiff–positive granules in the outer layer of eccrine ductal cells can be observed in:

A. Niemann-Pick disease

B. Lafora's disease

C. Fabry's disease

D. Hurler's syndrome

E. Kanzaki syndrome

87. Which of the following is the most common subset of findings in patients who have psoriatic arthritis
and ankylosing spondylitis with absence of IgM rheumatoid factor?

A. Polyarthritis

B. Arthritis mutilans

C. Distal interphalangeal arthropathy

D. Sacroiliitis

E. Extra-articular involvement of the lung, eye, or heart


88. Which of the following is not an expected complication with ablative laser skin resurfacing?

A. Milia

B. Acne

C. Ectropion

D. Rosacea

E. Contact dermatitis

89 . A consistent histopathologic feature of hydroxyurea dermopathy is:

A. Hydropic degeneration

B. Leukocytoclasis

C. Eosinophilic spongiosis

D. Pigment incontinence

E. Fibrinoid necrosis of vessel walls

90. Which of the following is the precipitating agent in acquired porphyria cutanea tarda?

A. Alcohol

B. Estrogen

C. Polychlorinated hydrocarbon

D. Iron

E. All of the above

91. Viral capsid antigen (VCA) IgG is positive in acute infection with:

A. Herpes simplex.

B. Condyloma acuminata.

C. Infectoius mononucleosis.

D. Reseola infantum.

E. Rocky Mountain spotted fever.


92. Blueberry muffin lesions are seen in:

A. TORCH syndrome.

B. Toxoplasmosis.

C. Kaposi sarcoma.

D. Varicella.

E. Fifth disease.

93.dew drop on a rose petal" is seen in:

A. Pityrisis resea.

B. PLEVA.

C. Insect bite.

D. Folliculitis.

E. Chicken pox

94. Herpes zoster vesicles on the little finger of the hand indicates involvement of dermatome :

A. C2.

B. C8.

C. C6

D. C5

E. T1.

95. The initial response to diphencepron (DPCP) is usually seen after:

A. 12 weeks.

B. 2 weeks.

C. 24 weeks.

D. 6 weeks.

E. 4 weeks.
96. Livido racemosa is generally indicative of:

A. Polycythemia vera.

B. Sneddon's syndrome.

C. Cutaneous polarthritis nodosa.

D. Raynaud's disease.

E. Cryoglobulinemia.

97. Mebendazole is not used in the treatment of :

A. Ascariasis.

B. Toxocariasis.

C. Loiasis.

D. Ankylostomiasis.

E. Enterobiasis.

98. Which of the following is not a histologic feature of telogen effluvium :

A. Normal total number of hairs.

B. Increase in telogen count to a greater 20%.

C. Increased dermal mucin is often present.

D. Abscence of inflammation.

E. Normal number of terminal (large) hairs.

99. Which of the following is not true about infantile hemangioma:

A. It is the most common tumor in infancy.

B. Majority of lesions are seen within the first weeks of life.

C. Its more common in males.

D. It appears more often in premature infants.

E. Sever complicated hemangiomas have been observed more frequently in girls.


100. The most common type of melanoma in patient with darker skin types is:

A. Superficial spreading melanoma.

B. Lentigo maligna melanoma.

C. Acral lintiginous melanoma.

D. Nodular melanoma.

E. Melanoma in situ.

101. Which of the following drugs does not cause flushing?

A. Calcitonin.

B. Hydralazine.

C. Methotrexate.

D. Seldinafil.

E. Opiates.

102. Which of the following has the longest plasma half-life?

A. Hydrocortisone.

B. Prednisolone.

C. Betamethasone.

D. Triamcinalone.

E. Methylprednisolone.

103. Blue-gray discoloration of the skin is a side effect of:

A. Azathioprine.

B. Hydroxychlorquine.

C. Mycophenolate mofetil.

D. Cyclosporine.

E. Hyroxyuria.
104. The treatment of choice for porphyria cutanea tarda is:

A. Erythropiotin.

B. Titanium dioxide.

C. Phlebotomy.

D. Antimalarial.

E. Warfarin.

5. β i roglo uli a loidosis is:

A. Myeloma - associated.

B. Tumor - associated.

C. Nodular amyloidosis.

D. Dialysis – associated.

E. Lichen amyloidosis.

106. SLE rate is highest in patient with a deficiency of:

A. C3.

B. C1q.

C. C5.

D. C2.

E. C9.

107. "Botton hole" sign is seen in:

A. Cutis laxa.

B. Striae distensae

C. Anetoderma.

D. Pseudoxanthoma elasticum.

E. Porphyria cutanea tarda.


108. Lymphadenopathy is not a manifestation of :

A. Syphilis .

B. Chancroid .

C. Lymphogranuloma venereum .

D. Granuloma inguinale .

E. Herpes genitalis.

109. The most specific antinuclear pattern for SLE is:

A. Speckled patter .

B. Homogendus pattern .

C. Rim pattern .

D. Nucleolar pattern .

E. Fine speckled pattern

110. One of the following histological features is not seen in pretibial myxedema:

A. Epidermis is usually affected.

B. Wide separation of collagen bundles by acid mucopolysaccharides .

C. Mast cells markedly increased in mucinous areas .

D. Hair follicles not destroyed .

E. Perivascular inflammatory infiltrate a round dilated blood vessels

111. When staphylococcal scalded skin syndrome occurs in adults , it is often associated with which of
the following pre-existing:

A. Complement deficiencies.

B. Liver failure.

C. Renal insufficiency .

D. Scarlet fever.

E. Rheumatic fever.
112. Histological examination of nevus anemicus shows:

A. Presence of melanocytes around blood vessels.

B. Absence of melanocytes.

C. Smooth muscle hamartoma.

D. Normal skin.

E. Collections of mast cells.

113.

114. The following is not true regarding paraneoplastic pemphigus

A. May associated with Castleman disease .

B. Most often related to a leukemia or non - Hodgkin lymphoma

C. Cutaneous lesions highly variable .

D. 25% have mucosal and lingual ulceration.

E. Positive IIF testing with monkey esophagus does not distinguish it from pemphigus vulgaris.
115. A patient with multiple sebaceous adenomas should be screened and his first - degree relatives
with one of the following examination:

A. CT scan of the chest .

B. Colonoscopy .

C. Retinal examination .

D. Mammogram .

E. Renal ultrasound

116. A gluten - free diet , used to manage dermatitis herpetiformis can safely include product derived
from:

A. Barley .

B. Rice .

C. Rye .

D. Triticale .

E. Wheat .

117. The diagnostic technique useful for heritable blistering skin diseases is :

A. Immunomapping .

B. Immunoprecipitation .

C. ELISA.

D. Immunoblotting .

E. Fluorescent-DT-PCR.

118. In distinguishing alopecia areata from trichotillomania under the microscope, which of the
following features favors alopecia areata?

A. Multiple catagen hairs

B. Pigment casts

C. Granulomatous inflammation

D. Eosinophils

E. Follicular plugging
119. Which of the following is the most common cause of death in malignant atrophic papulosis (Degos
disease):

A. Cerebral infarction.

B. Renal insufficiency.

C. Myocardial disease.

D. Peritonitis.

E. Cutaneous ulcers with sepsis.

120. Which of the following is true regarding subacute cutaneous lupus erythematosus:

A. The Ro/ SS-A autoantibodies is the characteristic marker

B. Patients can have both psoriasiform and atrophic plaques .

C. The face is commonly affected.

D. Imunoglobulins and complement deposition in papillary dermis.

E. Corticosteroids represent first –line systemic therapy.

121. When treating a male patient with methotrexate, current recommendation a to avoid conception:

A. During therapy only.

B. During therapy and 3 months after discontinuation.

C. During therapy and 6 months after discontinuation.

D. During therapy and 1 year after discontinuation.

E. During therapy and 2 years after discontinuation.

122 . The most common form of psoriatic arthritis is :

A. Distal interphalangeal arthritis.

B. Arthritis mutilans.

C. Spondylitis.

D. Arthritis indistinguishable from rheumatoid arthritis.

E. Asymmetric oligoarthritis
123. The findings of facial edema that progresses to purpuric lesions of the face and extremities in a
young child without systemic involvement is most consistent with a diagnosis of:

A. Wegener's granulomatosis.

B. Finkelstein's syndrome.

C. Churg-Strauss syndrome.

D. Mucocutaneous lymph node syndrome.

E. Takayasu's disease.

124. In distinguishing alopecia areata from trichotillomania under the microscope, which of the
following features favors alopecia areas? except

A. Multiple catagen hairs

B. Pigment casts

C. Granulomatous inflammation

D. Eosinophils

E. Follicular plugging

:‫هذه موجودا ال يستولوجي ع دة‬The most characteristic feature is a peribulbar lymphocytic infiltrate, which is
described as appearing similar to a swarm of bees. The infiltrate often is sparse and usually involves only
a few of the affected hairs in a biopsy specimen. Occasionally, no inflammation is found, which can
result in diagnostic difficulties. A significant decrease in terminal hairs is associated with an increase in
vellus hairs, with a ratio of 1.1:1 (normal is 7:1). Other helpful findings include pigment incontinence or
the presence of eosinophils in the follicular stellae, multiple catagen hairs, hyperkeratosis of the
infundibulum, and pigment casts in the infundibulum

. : ‫ ي ون الجوا‬: ‫ يجب أن ت ون كل م يلي صحيح عدا‬، ‫أعتقد أن صيغ السؤا خطأ‬


5 An eight-year-old female presents with palpable purpura of the extremities, abdominal pain, and
microscopic hematuria of one week's duration. Classic immunopathology of the cutaneous lesions
would show:

A. IgA and C3 at the basement membrane.

B. Diffuse fibrin at the basement membrane.

C. IgA associated with leukocytoclastic vasculitis.

D. IgG associated with leukocytoclastic vasculitis.

E. IgG and C3 associated with leukocytoclastic vasculitis.

126. The most significant systemic manifestations of Kawasaki disease are related to the involvement of:

A. Cerebellar arterioles.

B. Coronary arteries.

C. Glomerular capillaries.

D. Peripheral lymphatic vessels.

E. Pulmonary venules.

127. Subcorneal blistering, acantholysis and neutrophilic infiltrate are characteristic for:

A. Linear IgA dermatosis.

B. Pemphigus herpitiformis.

C. Epidermolysis bullosa aquisita.

D. Paraneoplatic pemphigus.

E. Pemphigus vegitans.
128. Mickulicz cells are characteristic of :

A. Glanders.

B. Erysipeloid.

C. Erlichiosis.

D. Rhinoscleroma.

E. Granuloma inguinale.

129 . Distal blistering dactylitis is associated with which causative organism?

A. Pseudomonas.

B. Staphylococcus..

C. Pityrosporum.

D. Streptococcus.

E. Candida albicans.

130. The most significant systemic manifestations of Kawasaki disease are related to the involvement of:

A. Cerebellar arterioles.

B. Coronary arteries.

C. Glomerular capillaries.

D. Peripheral lymphatic vessels.

E. Pulmonary venules.

131. Which of the following is not a feature of Gianotti Crosti syndrome?

A. Symmetry of the lesions.

B. Involvement of the face, buttock and legs.

C. Duration of at least 10 days.

D. Extensive trunk lesions.

E. Monomorphic papulovesicular lesions.


. The principal cause of morbidity and mortality in patients with CREST syndrome is:

A. Sepsis.

B. Myocardial infarction.

C. Pulmonary hypertension.

D. Acute renal failure.

E. Chronic renal failure.

134. This he most frequently reported skin eruption in mycoplasma infection is:

A. Urticaria.

B. Erythema multiformis.

C. Gianotti Crosti syndrome.

D. Erythema nodosum.

E. Exanthema.

135.The most common early symptom in melanoma is:

A. Tendernes

B. Pruritus

C. Bleeding

D. Ulceration

E. Elevation

136. One of the following drug commonly causes linear IgA dermatosis?

A. Hyroxyuria.

B. Vancomycin.

C. ACE inhibitors.

D. Hydralazine.

E. Bromides.
137. Of the following the most common manifestation of tuberous sclerosis is:

A. Cardiac arrhythmias

B. Mental retardation

C. Seizures

D. Enamel pitting

E. Visual loss caused by retinal astrocytoma

138. About necrobiosis lipoidica which of the following is false:

A. Trauma is a risk factor.

B. It is always associated with uncontrolled diabetes mellitus.

C. Ulceration of necrobiosis lipoidica is painful.

D. Granuloma annulare is a differential diagnosis..

E. Histopathologically shows palisading granuloma.

139 . The mechanism of action in Q- switched lasers is:

A. Tissue evaporation.

B. Vascular rupture.

C. Water boiling.

D. Photothermolysis.

E. Melanocyte destruction.
140 . In familial benign pemphigus which of the following statements is not correct:

A. It is an autosomal dominant.

B. Onset is usually in early childhood.

C. Often mistaken as intertrigo.

D. Histologically shows acantholysis throughout the epidermis.

E. There is defect in adhesion complex between desmosomal proteins and tonofilaments.

141. Which of the following is false in tuberous sclerosis?

A. Hypomelanotic macules present at birth.

B. Pariungul fibroma usually arise in late childhood.

C. Heart rhabdomyoma can be associated with the disease.

D. Ash-leaf appearance is the only form of hypomelanotic macules.

E. Equal incidents in males and females.

142. Which of the following is not correct about X- linked ichthyosis?

A. The face is spared.

B. Corneal opacities can affect carriers.

C. Associated with transglutaminase deficiency.

D. It is characterized by dirty brown scales.

E. Can be associated with cryptochidism.

143. Which of the following statements about intrahepatic cholestasis of pregnancy is false?

A. It typically presents with generalized pruritus in the last trimester.

B. May or may not be associated with clinically detectable jaundice.

C. Liver transaminases are usually markedly elevated.

D. Is associated with high incidence of stillbirth.

E. Recurrence of itching may occur in any subsequent pregnancy.


144. In paraneoplastic pemphigus which of the following is false:

A. Combine features of pemphigus vulgaris & erythema multiforme.

B. Anti bullous pemphogiod antigen type I is detected.

C. Anti desmoplakin I is detected.

D. Skin is primarily involved.

E. Management is by treatment of malignancy & may also require steroids.

145. About reactive perforating collagenosis which of the following is true:

A. It is transepidermal elimination of altered elastic tissue.

B. Usually starts in old age .

C. Trauma is a trigger factor.

D. Face is most commonly affected.

E. It is caused by coxsakiervirus A 16.

146. Which one of the following is not true in miliaria rubra?

A. Usually pruritic

B. Typically occurs on the upper trunk and neck

C. Characterized by small, follicular, erythematous macules and papules

D. Normally sterile on culture

E. Initial treatment requires placement of the patient in a cool environment

147. . Which one of the following is true regarding microcomedo formation?

A. Keratohyaline granules are increased in size

B. Keratohyaline granules are decreased in number

C. Lamellar granules are increased

D. Tonofilaments are increased

E. e. decreased intercellular cohesiveness of the corneocytes


148. Which one of the following is the most common site for Osteolytic bone lesions in Acne fulminans?

A. Humerus

B. Iliosacral joints

C. Ankles

D. Clavicle

E. Ribs

149. Among the following, which is the commonest cause of conjunctival involvement

A. Osteomas

B. Epidermoid cysts

C. Hemangiomas

D. Duodenal and colonic polyps

E. Abormalities of the teeth

150 . Which one of the following diseases has Ragged cuticles?

A. Vitiligo

B. Dermatomyositis

C. Herpis infection

D. Epidermolysis bollosa simplex

E. Dowling–Meara syndrome

151. Among the following lesions, which is the most likely to turn malignant?

A. Nevus comedonicus

B. Verrucous epidermal haevus

C. Naevus sebaceous

D. Sebaceous adenoma

E. Steatocytoma multiplex
152 . Which one of the following is the most specific antinuclear pattern for CREST syndrome?

A. Rim

B. Speckled

C. Homogenous

D. Nucleolar

E. Centromeric

153. Which one of the following diseases has matted telangiectasias?

A. Hereditary hemorrhagic telangiectasia

B. Eosinophilic fasciitis

C. Limited Systemic sclerosis

D. Stiff skin syndrome

E. Nephrogenic systemic fibrosis

154. Which one of the following is not a feature of anti-synthetase syndrome?

A. Acne

B. Erosive polyarthritis

C. Fever

D. Interstitial lung disease

E. Raynaud's phenomenon

155. . Which one of the following is not a feature of reactive arthritis (Reiters syndrome)?

A. Gyrate white plaques on the penis

B. Shiny patches on the palate

C. Anterior uveitis

D. Trachyonychia

E. IgA nephropathy
156. In which one of the following diseases the activity of 2,3-dioxygenase is increased?

A. Involuting phase of hemangiomas

B. Proliferating phase of hemangiomas

C. Normal vascular development

D. Normal lymphatic development

E. Resting phase of hemangiomas

157. Which one of the following is true regarding Junction lines?

A. Are skin tension line

B. Refer to the normal skin-graft interface of a grafted defect

C. Are difficult to ascertain in most patients

D. Separated cosmetic units

E. Not useful for tissue matching

158. Which one of the following is not an important risk factor for arterial Ulcers?

A. Cigarette smoking

B. Age >40 years

C. Hyperlipidemia

D. Female

E. Hyperhomocysteinemia

159. The infraorbital foramen is located:

A. 5 cm below the infraorbital rim in the midpupillary line

B. 1 cm below the infraorbital rim in the midpupillary line

C. 2 cm below the infraorbital rim in the midpupillary line

D. 3 cm below the infraorbital rim in the midpupillary line

E. 4 cm below the infraorbital rim in the midpupillary line


160. Which one of the following is NOT a predisposing factor to colonization and infection with MRSA?

A. Age (older than 65)

B. Prior antibiotic therapy

C. Recent hospitalization

D. Vaccination

E. Chronic illness.

161. Which one of the following is NOT true in rhinoscleroma bacillus?

A. It is a gram-positive rod

B. It is best visualized with the Warthin-Starry silver stain.

C. It is always encapsulated

D. It is Non motile organism

E. The bacilll are found within (Mikulicz cells)

162. A 5-years old boy had Impetigo which was complicated by acute glomerulo-nephritis, which one of
the following is nephrotogenic strains of streptococcus?

A. 7

B. M-type 2

C. M-type 9

D. 21

E. 25
163. Anesthesia of the involved skin is very characteristic in:

A. Cellulitis

B. Chronic Lymphangitis

C. Erysipelas

D. Ecthyma

E. Necrotizing fasciitis

164. A 4-year-old boy has radiographically proven osteomyelitis following a cat bite to the finger. A
Gram stain reveals pleomorphic gram-negative rods. The most likely diagnosis is:

A. Bartonella henselae.

B. Francisella tularensis.

C. Pasteurella multocida.

D. Haemophilus influenzae.

E. Bacteroides fragilis

165. Which one of the following granules is seen in Botryomycosis discharge?

A. Sulfur granules

B. Red granules

C. Black granules

D. White granules

E. Blue granules
166.

167. A 35-years old man, has six purple nodules in a linear distribution after a minor injury. The nodules
were moderately painful. Tissue culture showed Cigar-shaped hyphae. The most likely diagnosis is:

A. Cryptococcosis

B. Sporotrichosis

C. Blastomycosis

D. Histoplasmosis

E. Coccidioidomvcosis

168. A 22-year-old male has firm, raised, red, non-tender chancre of the genitalia. Which one of the
following is the most appropriate course of action for the physician?

A. Swab the chancre and culture on Thayer-Martin agar

B. Swab the chancre and perform a Gram stain

C. Perform a dark-field examination on a swab of the active lesion

D. Swab the chancre and culture on blood ager


E. Skin biopsy

169. which one of the following treatments is the best method to treat ulnar nerve abscess in patients
with leprosy ?

A. High doses of steroid

B. Incision and drainage

C. Thalidomide

D. High dose of clofazimine

E. NSAIDs

170. Which one of the following tests is good indicator to assess the progress of HIV-positive patient to
AIDS?

A. CD4 lymphocyte count

B. HIV antibody test

C. HIV RT PCR

D. Neopterin

E. HIV p24 antigen

171. Which one of the following is not causing Visceral leishmaniasis?

A. L.donovani

B. L.tropica

C. L.chagasi

D. L. longipaipis

E. L.infantum
172. A 5 month-old boy has ivory color of the hair and his investigations showed low serum copper and
copper-dependent enzymes. Which one of the following is the most likely diagnosis:

A. Menkes Kinky Hair Syndrome

B. Naxos' disease

C. Carvajal syndrome

D. Laron syndrome

E. Anorexia nervosa

173. Mutation in loricrin occurs in:

A. Vohwinkel syndrome

B. Ichthyosis vulgaris

C. Erythrokeratoderma variabilis

D. Olmsted syndrome

E. Pachyonychia congenita

174. which one of the following diseases is characterized by an absence of lactoferrin?

A. Atopic dermatitis

B. Neutrophil-specific granule deficiency

C. Leukocyte adhesion defects

D. Di-george syndrome

E. Job syndrome
175. which One of the following is not a Tick-borne disease?

A. Tularemia

B. Relapsing fever

C. Rickettsial fevers

D. Lyme disease

E. Trichinosis

176. Most mutations are generated during:

A. Replication of damage RNA

B. Replication of damage DNA

C. Repair of damage mitochondria

D. Repair of damage golgi apparatus

E. Formation of cell wall

177. Which one of the following melanocortin receptors is highly expressed in melanocytes?

A. MC1R

B. MC2R

C. MC3R

D. MC4R

E. MC5R

178. Which one of the following diseases has excessive accumulation of glycosaminoglycans in the
elastic fibers?

A. Elastoderm

B. Cutis laxa

C. Pseudoxanthoma elasticum

D. Actinic elastosis

E. Marfan syndrome
179. Which one of the following diseases has a defects in ATP-dependent copper transport?

A. Homocystinuria

B. Osteogenesis imperfect

C. Menkes syndrome

D. Ehlers-danlos syndrome

E. Familial cutaneous collagenoma

180. Which one of the following is not a risk factor for actinic keratoses?

A. Age

B. Female gender

C. UV radiation

D. Immunosuppression

E. Phenotype of fair skin

181. Which one of the following diseases there is no risk to develop multiple basal cell carcinoma?

A. Nevoid basal cell carcinoma

B. Beneralized basaloid follicular hamartoma

C. Bazex syndrome

D. Rombo syndrome

E. Pachyonychia congenital type 11

182. Which one of the following diseases has Kamino bodies ?

A. Blue nevus

B. Spitz nevus

C. Halo nevomelanocytic nevus

D. Nevus spilus
E. Nevomelanocytic nevus

183. Which one of the following human papilloma virus is not linked to bowenoid papulosis?

A. HPV type 18

B. HPV type 31

C. HPV type 35

D. HPV type 39

E. HPV type 49

184. Which one of the following areas is the most common site for oral squamous cell carcinoma is?

A. Upper gingiva

B. Lower gingiva

C. Palate

D. Buccal mucosa

E. Floor of the mouth

185. Which one of the following stains has strong reaction in tumors with follicular differentiation?

A. Carcinoemberyonic antigen

B. Pancytokeratin

C. S-100 protein

D. Vimentin

E. Epithelial membrane antigen


186. Which one of the following is not a feature of Refsum s syndrome?

A. Deafness

B. Cardiomyopathy

C. Autosomal dominant

D. Ichthyosis in adulthood

E. Cerebellar ataxia

187. Which one of the following keratoderma diseases has focal keratoderma on pressure sites of the
hands and feet?

A. Papillon–Lefèvre syndrome

B. Mal de Meleda syndrome

C. Howel–Evans syndrome

D. Unna–Thost disease

E. Vörner disease

188. Which one of the following markers is very sensitive for dermatofibrosarcom protuberans?

A. CD34

B. CD44

C. Factor X111a

D. Apolipoprotein D

E. Hyaluronate

189. Which one of the following diseases has Elevated serum IgE?

A. Sjögren–Larsson syndrome

B. Lamellar ichthyosis

C. CHILD syndrome

D. Netherton syndrome

E. Refsum disease
190. Which one of the following diseases is charactarized by Dysfunction of SERCA2b protein?

A. Darier disease

B. Hailey–Hailey disease

C. Richner–Hanhart syndrome

D. Refsum syndrome

E. Netherton syndrome

191. Which one of the following clinical features is the classical presentation of Epidermolysis bullosa
acquisita (EBA)?

A. Inflammatory BP-like presentation

B. A non-inflammatory mechanobullous

C. Cicatricial pemphigoid -like presentation

D. Brunsting–Perry pemphigoid phenotype with scarring alopecia

E. Mucous membrane involvement

192. Which one of the following is a constant feature of paraneoplastic pemphigus?

A. Intractable stomatitis

B. Pseudomembranous conjunctivitis

C. Nasopharyngeal mucosal lesions

D. Penile mucosal lesions

E. Vaginal mucosal lesions

193. Which one of the following is not true regarding herpetiform pemphigus?

A. Most patients have a clinical variant of pemphigus foliaceus

B. Few patients have a clinical variant of pemphigus foliaceus

C. The target antigen is Dsg3 in most cases

D. Some patients will evolve into having pemphigus foliaceus


E. Characterized by eosinophilic spongiosis and subcorneal pustules with minimal or no apparent
acantholysis

194. Which one of the following diseases has Keratitis?

A. Erythrokeratodermia variabilis

B. Netherton syndrome

C. Refsum disease

D. KID syndrome

E. CHILD syndrome

195. Which one of the following is not true regarding Cicatricial pemphigoid (CP)?

A. The most frequently involved sites is the oral mucosae

B. The common oral presentation is erosive (desquamative) gingivitis

C. Conjunctival involvement may result in blindness

D. The esophageal mucosa involvement can be entirely asymptomatic.

E. 10% of patients with CP have in vivo bound autoantibodies directed against the BMZ of mucosae
and/or skin

196. Which one of the following drugs can cause Pseudoxanthoma elasticum-like changes?

A. Gold

B. Adalimumab

C. Penicillamine

D. Methotrexate

E. Cyclosporine
197. Erythrokeratodermia variabilis (EKV) is a disorder of cornification associated with:

A. Mutations in the connexin genes GJB3 and GJB4

B. Mutations in the filaggrin gene (FLG)

C. Somatic mutation

D. Hydroxysteroid-dehydrogenase deficiency

E. Mutations in K1 and K10

198. which one of the following diseases is characterized by Trichilemmomas?

A. Muir–Torre syndrome

B. Cronkhite–Canada syndrome

C. Degos disease

D. Peutz–Jeghers syndrome

E. Cowden disease

199. Which one of the following is a feature of Churg–Strauss syndrome ?

A. Neutrophilic infiltration

B. Eosinophilic infiltration

C. Lymphocytic infiltration

D. Mainly perifollicular giant cell infiltration

E. Mainly neutrophilic infiltration without necrosis

200. which one of the following is characterized by Necrolytic migratory erythema ?

A. Paraneoplastic pemphigus

B. Acquired hypertrichosis lanuginosa

C. Glucagon-secreting tumor of the pancreas


D. Erythema gyratum repens

E. Anti-epiligrin cicatricial pemphigoid

201. An 83 year old female has a biopsy of an ulcerated nipple lesion that is interpreted as Paget's
disease. A biopsy of the underlying breast tissue will most likely show which of the following?

A. Acute mastitis

B. Ductal carcinoma in situ

C. Intraductal papilloma

D. Invasive lobular carcinoma

E. Normal breast tissue

202. Which one of the following local anaesthetics is used in pregnancy?

A. Lidocaine

B. Mepivacaine

C. Etidocaine

D. Ropivacaine

E. Tetracaine

203.vOne of the followings is not correct about WHO classification of mastocytosis:

A. Type 1a cutaneous mastocytosis

B. Type 1b indolent systemic mastocytosis

C. Type II+ systemic mastocytosis with associated clonal hematological non-mast cell-linage disease

D. Type III+ aggressive systemic mastocytosis

E. Type IV B-cell leukemia


204. A four years old presented with fever for 5 days. On examination he has fever of 40°C, BP of
140/90, HR 100 bpm, RR 20/min. He has palpable cervical lymph nodes, a red tongue and a truncal rash
with desquamation of the hands and feet. What is the most likely diagnosis?

A. Infectious mononucleosis

B. Kawasaki disease

C. Measles

D. Parvovirus infection

E. Systemic lupus erythematosus

205. One of the following is not correct about EB atrophicans inversa:

A. Blistering occurs during the neonatal period.

B. The lesions affect chiefly the groin, perineum and axillae

C. Multifocal basal cell carcinoma in a non-healing site can occur

D. Healing may result is small, atrophic white streaks.

E. Dystrophic teeth, erosions at the cornea and feet and nail dystrophy are all features of disease

206. Increased VLDLs and eruptive type of xanthoma is observed in one of the following types of hyper-
lipoproteinemias:

A. Type I

B. Type II

C. Type III

D. Type IV

E. Type V
207. One of the followings is not correct about connective tissue panniculitis (CTP) and their
histopathology:

A. Morphea and scleroderma, septal panniculitis

B. Lupus panniculitis, lobular or mixed

C. Dermatomyositis, lobular or mixed

D. Annular atrophic CTP of the ankles, lobular or mixed neutrophic panniculitis

E. Atrophic CTP, lobular or mixed

208. One of the followings is not correct about Marfan syndrome:

A. Fibrillin 2 mutations are causative

B. An autosomal dominat disorder

C. Thin and slightly hyper extensible skin

D. Elastosis perforans serpiginosa

E. Striae atrophicae

209. One of the following is not correct about localized variants of lichen myxdemata:

A. Popular eruption

B. Plaques eruption

C. Mucin deposition with variable fibroblast proliferation

D. Monoclonal gammopathy

E. Absence of thyroid disorders

210. One of the following drugs is the least likely to cause morbilliform eruption

A. Opiates

B. Cocaine

C. Barbiturates

D. Amphetamines

E. Marijuana
211. One of the followings is not correct about Kasabach –Merritt syndrome:

A. It is a vascular tumor

B. Associated with a thrombocytopenic coagulopathy

C. This syndrome is more common in girls than in boys

D. Many of these tumors are noted at birth

E. Some patients may be successfully managed with vineristine

212. One of the following is not correct regarding vessel wall pathology as a cause of livedo reticularis :

A. Calciphylaxis

B. Sneddon s syndrome

C. SLE

D. Carbon dioxide arteriography

E. Cutaneous polyarteritis nodosa

213. One of the following chemicals is not associated with the development of non-melanoma skin
cancer:

A. Mineral oil

B. Coal tar

C. Psoralen ( plus UVA)

D. Nitrogen mustard

E. Salicylic acid

214. Dressing recommended for arterial type of ulcer is:

A. Foam

B. Hydrogel

C. Alginate

D. Hydrofiber
E. Charcoal

215. One of the following types of perforating diseases has very rare incidence and more common in
black women :

A. Reactive perforating collagenosis

B. Elastosis perfornans serpiginosa

C. Perforating folliculitis

D. Acquired perforating dermatosis

E. Perforating periumblical clcific elastosis

216.One of the following is not a benign vascular tumour

A. Infantile hemangiona

B. Cherry angioma

C. Hobnail hemangioma

D. Dabska-type hemangioendothelioma

E. Tufted angioma

217. Which of the following muscles should not be injected by Botulinum toxin if brow elevation is
requested by the patient?

A. Corrugator muscle

B. Depresson supercilli muscle

C. Procerus muscle

D. Medial part of frontalis muscle

E. Lateral part of frontalis muscle


218. One of the following cytomorphology is a criterion for histopathologic diagnoses of melanoma:

A. Signs of regression

B. Mitotic figures

C. Actinic elastosis

D. Asymmetry

E. Nets at base of lesions are occasionally large

219. One of the followings is not correct about warty dyskeratoma

A. Lesions grow slowly

B. They are most commonly located on the scalp, cheek, temple and forehead

C. Malignant degeneration has been reported

D. Most warty dyskeratomas are asymptomatic

E. It has been reported to coexist with SCC, and BCC.

220. One of the following tumours has location in mouth as major distinguishing features

A. Ameloblastoma

B. Sebaceous carcinoma

C. Mucinous carcinoma

D. Merkell cell carcinoma

E. Eccrine carcinoma

221. Suture 4-0 vicryl compared to 5-0 vicryl has

A. More tensil strength

B. Increased suture memory

C. Smaller suture diameter

D. Smaller needle
E. Increased knot security

222. Radial streaming pattern of pigmented lesion by dermoscopy indicates

A. Benign nevus

B. Pigmented basal cell carcinoma

C. Seborrheic keratosis

D. Melanoma

E. Has no indications

223. . Which of the following lasers has crystal medium ?

A. Ruby

B. Argon

C. Carbon dioxide

D. Copper vapor

E. Rhodamine dye

224. Gallstones most commonly occur in:

A. Erythropoietic protoporphyria

B. Hereditary coproporphyria

C. Variegate porphyria

D. Porphyria cutanea tarda

E. Acute intemittent prophyria

225. The specific test for the diagnosis of neurosyphilis is :

A. TPHA

B. MHA-TP

C. SPHA

D. FTA-ABs
E. FTA-ABs 19s-IgM

226. One of the followings is not correct about arthritis – dermatosis syndrome ( gonococcemia)

A. The risk factor is menstruation

B. With a longer duration of symptoms, positive blood cultures become more common

C. Deficiencies in the late complement components ( C5-C9)

D. Gonococcal tenosynovitis affects primarily larger joints

E. The cutaneous lesions consist of scattered necrotic pustules due to embolic septic vasculitis

227. One of the followings is not correct about neoplastic HIV-related cutaneous disorders

A. The SCC:BCC ratio is not changed in those with HIV infection

B. Leiomyomas and leiomyosancoma are seen more frequently in HIV- infected perdiatric patients

C. Lymphomas of both B and T-Cell lineage may develop in HIV-infected adults and children

D. Kaposi s sarcoma can be seen at any stage of HIV infection

E. HIV-infected children have higher incidence of mucosa-associated lymphoid tissue (MALT) lymphomas
involving pulmonary and gastric mucosae

228. Which of the following lasers has the shortest depth of penetration ?

A. KTP

B. Ruby

C. CO2

D. Excimer

E. Erbium : YAG
229. About basal cell carcinomas ( BCCs) in the perianal and genital region, one is not correct

A. Are rare, comprising less than 1% of BCCs

B. Usually exhibit an association with human papillomavirus type 16 or 18

C. Maybe associated with previous exposure to therapeutic radiation

D. May occur in patients who have basal cells nevus syndrome

E. Are associated with HPV types 6,11

230. The pseudo-darier signs occurs in :

A. Smooth muscle hamartoma

B. Mastocytoma

C. Neurofibroma

D. Nevoid hypertrichosis

E. Hemihypertrophy

231. Acquired hypertrichosis lanuginosa is not associated with :

A. Lung cancer

B. Breast carcinoma

C. Plamoplanter keratoderma

D. Colon cancer

E. Brain tumor

232. One of the followings is not correct about bioengineered immunomodulators

A. Alefacept and efalizumab are interfere with T-cell activation

B. Anakinra is an agent that binds to IL-10 receptors

C. Rituximab is an agent that blocks CD2o on B.cells

D. Rituximab is approved for treatment B-cell lymphoma


E. Agents that block TNF-α are eta er ept, i lfi i a a d adali u a

233. The major chromophore in skin for the generation of single oxygen is:

A. DNA

B. Keratin

C. Urocanic acid

D. Selenium

E. Collagen

234. The major cause of mortality in patient with calciphylaxis is:

A. Cerebrovascular accident

B. Sepsis

C. Arrhythmia

D. Diabetic coma

E. Myocardial infarction

235. Which of the following statements is true?

A. Calcitonin gene-related peptide and substance P are increased in the alopecia areata (AA) scalp

B. Calcitonin gene-related peptide and substance P are decreased in AA scalp

C. I terleuki s α a d β i hi it hair gro th i itro

D. Tumor necrosis factor alfa enhances hair growth in culture

E. B and C

236.Menkes kinky hair syndrome

A. Is x-linked dominant

B. Is not commonly associated with CNS in pairment

C. Has been related to a defect in intes- iron transport

D. Exhibits decreased free sulfydryl – content of hair

E. Is associated with pilitorti, monilethrix and trichorrhexis nodosa


237. Papular urticaria most often affects patient in which age group of the following:

A. Less than 2 weeks old

B. 1 month to 6 months old

C. 1 year to 2 years old

D. 2 years to 7 years old

E. 7 years to 18 years old

238. A 6 month-old infant is presented with linear atrophic bands following blaschko s lines, with
scattered telangiectases, several soft reddish-yellow nodules and lobster claw type deformity of her
left hand, you suspect that she has :

A. Cornelia de langer s syndrome

B. Rubinstein-taybi symndrome

C. Omenn s syndrome

D. Sweet s syndrome

E. Marfan s syndrome

F. Goltz syndrome

239. Angio leiomyomas occur most frequently on

A. Head and neck

B. Acral

C. Trunk

D. Upper extremities

E. Lower extremities
240. The Brunsting- perry variant of cicatricial pemphigoid:

A. Affects the trachea and larynx

B. Is a non-scarring variant affecting only the skin

C. Is localized to the head and neck

D. Usually results in blindness

E. Always affects some mucosal surface

241. Which of the following types of melanoma arise from dermal melanocytes:

A. Superficial spreading melanoma.

B. Spitz-nevus (Juvienile melanoma).

C. Mucosal melanoma.

D. Melanoma in giant hairy nevus.

E. Melanoma in nail matrix.

242. Which of the following will most reliably distinguish pruritic urticarial papules and plaques of
pregnancy from herpes gestationis?

A. Location of the lesions on the body

B. Gestational age of the patient at the onset of the eruption

C. Results of direct immunofluorescence

D. Eosinophil count

E. The presence or obsence of vesicles

243. Cutaneous manifestations of antiphospholipid antibody syndrome does not include one of the
following:

A. Livedo reticularis

B. Ulcers

C. Purpura

D. Calcinosis
E. Necrosis

244. Which type of Ehlers-Danlos syndrome has ocular involvement including scleral fragility, retinal
detachment, blue sclera and intraocular hemorrhage?

A. Type I, classic

B. Type III, hypermobility

C. Type IV, vascular

D. Type VI, kyphoscoliosis

E. Type VIIC, Dermatosparaxis

245. Anti-Jo-1 antobodies are directed against

A. Topoisomerase

B. Gyrase

C. Histidyl transfer RNA synthetase

D. Phospholipase

E. Lysyl oxidase

250. The worst prognostic factor in Steven Johnson syndrome is:

A. Low PVC.

B. Low neutrophils.

C. Increased WBC.

D. High ESR.

E. Microalbuminemia.

251. One of the following is not correct regarding mycophenolate mophetil :

A. Mycophenolic acid is reactivated by B-GIU- curonidase in the epidermis

B. Selectively inhibit the enzyme inosine monophosphate dehydrogenase.

C. The most common side effect is GI symptoms.

D. It is indicated for both pemphigus vulgaris and psoriasis.


E. Both cyclosporine and tacrolimas are increasing mycophenolic acid level

252. Nephrotoxicity and genital ulcers are side effects of one of these :

A. Valacyclovir.

B. Gancyclovir.

C. Famcilovir.

D. Foscarnet.

E. Acyclovir.

253. One of the following is not a feature of Lidocaine toxicity:

A. Ototoxicity.

B. Nystagmus.

C. Seizure.

D. Slurred speech.

E. Perioral numbness.

254. One of the following is FALSE about herpes gestationis:

A. Onset during second trimester.

B. Severe oral involvement.

C. Starts around umbilicus.

D. The antigen is bullous pemphigoid antigen 2 (Bp Ag2).

E. Eosinophilic subepidermal blister.

255. The drug of choice for neurosyphilis is:

A. Third generation cephalosporine.

B. Procaine penicillin.

C. Crystalline penicillin.

D. Benzathine penicillin.

E. Azithromycin.
256. One of the following is not a feature of Acquired cold urticaria:

A. Presence of itchy weal.

B. Flare up of lesions after cold exposure.

C. Absence of angio-odema.

D. Lesions typically confined to cold exposed skin areas.

E. The lesions usually disappear within 30-60 minutes to several hours if cold is removed.

257. One of the following is not specific HIV-associated disorders when CD4 < 500 cells /mm³:

A. Oropharyngeal condidiasis.

B. Herpes zoster.

C. Seborrheic dermatitis.

D. Eruptive atypical melanocytic nevi and melanoma.

E. Kaposi s sarcoma.

258. Granular parakeratosis is seen mostly in:

A. Genitalia of adult male.

B. Scalp of infant.

C. Axilla of adult female.

D. Leg of adult female.

E. Chest of adult male.

259. One of the followings is not correct about dyskeratosis congenita:

A. Brittle, sparse hair.

B. Nail dystrophy.

C. Oral leukoplakia.

D. Reticulated hyperpigmentation.

E. Fanconi type pancytopenia.


260. Which of the following is not an action of cyclosporine therapy?

A. Inhibition of microphage inhibition migration.

B. Decrease synthesis and release of IL-1 by Mycrophage.

C. Decrease synthesis and release of IL-2.

D. I hi itio of δ i terfero produ tio

E. Inhibition of suppresor T cells

261.One of the followings is not correct about congenital melanocytic nevus:

A. It is melanocytic nevi present at birth.

B. They may or may not have associated with hypertrichosis, and perifollicular hypo or
hyperpigmentation may be seen.

C. Satellite congenital melanocytic nevi often accompany of giant congenital melanocytic nevi.

D. All sizes of this nevus may be associated with melanoma.

E. Neurocutaneous melanosis associated with small congenital melanocytic nevi

262 . One of the followings cytokines is not elevated in langerhans cell histocytosis:
A. TNF-σ .

B. Interferon-δ.

C. Granulocyte- monocyte colony-stimulating factor (GM-CSF).

D. IL-LO.

E. TNF

263. One of the following types of B.C.C has high recurrence rate:

A. Micronodular BCC.

B. Fibroepithelioma of pinkus.

C. Nodular BCC.

D. Superficial BCC.
E. Morpheaform BCC.

264. Prolyl hydroxylation reaction is the initial step in the biosynthesis in WHICH of the following :

a. Collagen

b. Elastin

c. Keratin sulphate

d. Hyaloronic acid

e. Keratin

265. . A Patient with anaemia, glossitis and necrolytic skin lesions most likely has

A. Sarcoidosis

B. Lupus erythematosus

C. Glucagonoma

D. Hypernephroma

E. Bronchogenic carcinoma

266. . Which one of the following dermatophytes causes endothrix type of hair invasion?

A. Microsporum audouinii

B. Microsporum canis

C. Trichophyton verrucosum

D. Trichophyton mentagrophytes

E. Trichophyton violaceum

267. One of the following conditions does not cause elastolysis (cutis laxa)

A. Blepharochalasis

B. Secondary anetoderma

C. Chronic atrophic acrodermatitis

D. Osteogenesi imperficta

E. Granulomatous slack skin


268. Adiposis dolorosa (dercum disease) is:

A. Lobular panniculitis

B. Localized lipoatrophy

C. Factitialpanniculitis

D. Painful lipomatosis

E. Hibernoma

269. are seen in KOH preparation from an infection of:

A. Horteawerneckii (Exophiala werceckii pheoannello myces werneckii)

B. Piedraia hortae

C. Trichosporon beigelii complex of species

D. Scopulariopis brevicaulis

E. Scytalidium dimidiatum

270. In which one of the following disorders antibiotics ( Tetracycline and Amoxicillin) are used?

A. Follicular atrophoderma

B. Elastoderma

C. Chronic atrophic acrodermatitis

D. Vermiculate atrophoderma

E. Atrophoderma of pasini and pierini

271. One of the following drugs does not cause secondrary Raynaud's phenomenon:

A. Imipramine

B. Bleomycin

C. Amphetamines

D. Oral countraceptive

E. Nifidipine
272. One of the following is not criterion for the diagnosis of primary Raynaud's phenomenon:

A. Intermittent attacks

B. Bilateral distribution

C. Female under the age of 25 years

D. History of cold intolerance childhood

E. Abnormal nail fold capillaries

273. About cellulite, one of the following statements is True:

A. Hypertrophy and hyperplasia of adipocytes are major aetiological factors in the development of
cellulite

B. Pregnancy improves cellulite

C. African women are less prone to cellulite than white women

D. Cellulite is evident only after skin compression or muscle contraction

E. Mesotherapy is an effective treatment

274. . About cold related skin injuries which one of the following statements is TRUE?

A. Freeze induced lesions results from denaturing of the cellular proteins

B. Rapid immersion in warm water of about 40 c for 20 minutes is recommended for the treatment of
trench foot

C. Perniosis (chilblains) results from abnormal reaction to cold

D. Acrocyanosis is an ulcerative lesion

E. Erythrocyanosis commonly involves


275. In one of the following sites, injuries should be considered not-accidental in children?

A. Medical side of the thigh

B. Front of leg

C. Knee

D. Bony

E. Forearm

276. Which one of the following statement is not true?

A. Kyrle's disease is a primary perforating dermatosis

B. Pyronie's disease is a penile fibromatosis

C. Systemic hyalinosis is a cause of infantile stiff skin

D. Ainhum is a bony outgrowth of the distal finger joint

E. Pangeria is an accelerated ageing process starting after puberty

277. Mixed type of panniculitis is:

A. Neutrophilic panniculitis

B. Lupus profundus

C. Relapsing febrile nodular panniculitis(Weber Christian syndrome)

D. Idiopathic nodular panniculitis

E. Pancreatic

278. One of the following is used for the treatment of senile pruritus:

A. Hydrocortisone

B. White soft paraffin

C. Topical diphenydramine

D. Potassium permanganate (at a concentration of 1:10000)

E. Benzyl benzoate 25%


279. Itching localized in distribution occurs in:

A. Iron deficiency

B. Polycythaemia vera

C. Thyroid disease

D. Uraemia

E. Diabetes mellitus

280. Which one of the following description refers to anetoderma?

A. Linear streaks of atrophy and telangiectasia with the subcutaneous fat immediately located beneath
the epidermis

B. Circumscribed areas of slack skin with loss of elastic tissue on histopathology

C. Yellow thick wrinkled skin with elastotic degeneration

D. Cigarette paper – like scars and thin waekly polarized collagen

E. Pitted and scared skin with little changes on light microscopy

281. One of the following is non- acute porphyria:

a. Acute intermittent porphyria

b. Variegate porphyria

c. Hereditary coproporphyria

d. PCT

e. ALA-D deficiency porphyria

282. One of the following is not correct about cockayne syndrome (cs):

a. Photosensitivity with pigmentary changes

b. Loss of adipose tissue

c. Hypogonadism

d. Classification of basal ganglia


e. CS type III, late onset normal growth and development

283. One of the following systemic Treatment of morphea has level 1 of evidence:

a. Penicillin

b. Vit D analogues

c. Methotrexate

d. Corticosteroids

e. Cyclosporine

284. Dermoscopically, a globular pattern with hyphae-like structures is characteristic for:

a. Recurrent melanocytic nevus

b. Dermal nevus

c. halo nevus

d. Atypical melanocytic nevus

e. Medium – sized congenital nevus

285. One of the following is not correct about Flegel disease:

a. Popular lesions are most commonly observed on the dorsal aspect of the feet and distal arms and legs

b. Most lesions are symptomatic

c. Patient occasionally complain of pruritus

d. The dis has been reportedly associated with DM and hyperthyroidism

e. Treatment by PUVA with calcipotriol

286. Bacilli in skin lesion are not detected in the following type of leprosy:

a. Lepromatous leprosy

b. Borderline Lepromatous leprosy

c. Mid - Borderline leprosy

d. Borderline tuberculoid leprosy

e. Tuberculoid leprosy
287. One of the following is not correct about plane xanthoma and xanthelasma:

a. Younger patients with xanthelasma are haven t an underlying lipid disorder

b. Intertriginous plane xanthoma may occur in the antecubital fossae or the web spaces of the finger

c. Xanthoma striatum palmare are almost dg for dysbetalipoproteinemia

d. Plane xanthoma of cholestasis may occur as a complication of atresia or primary biliary cirrhosis

e. Plane xanthoma can occur in anormolipemic patient, where they may signal the presence of an
underlying monoclonal gammapathy

288. One of the following is not correct about post – steroid panniculitis.:

a. onset 1 – 40 days after cessation of high- dose systemic cortico steroid therapy

b. firm red plague on chest, arms, trunk are observed

c. the associated systemic finding are leukemia, cerebral edema and nephrotic synd

d. histology is a mixed lobular- septal panniculitis

e. patient characteristics is a children 1 – 14 y old

289. One of the following markers is( – ) in blood vessels :

a. Laminin

b. Collagen type XVIII

c. CD34

d. VEGFR-2

e. VEGFR-3

290. One of the following is not clinical presentation of talc foreign body reaction:

a. Sarcoide- like papules

b. Furuncles

c. Thickening and erythema of an old scar

d. Umbilical stumps

e. Pyogenic granuloma-like
291 . Which of the following is a major diagnostic criteria for nevoid BCC ?

a. Macrocephaly

b. Congenital Malformation

c. Bilateral Ovarian Fibrom

d. 3 or more palmar or planter pits

e. Medulloblastoma

292. One of the following is incorrect about poroma:

a. Usually present as solitary papules, plaques or nodules

b. When sessile vascular plaques surrounded by thin indented moats appear on the palms and soles, an
astute dermatologist can strongly suspect the dg

c. The scalp is a rare site for poroma

d. Occasionally, a poroma will be pigmented

e. Rarely, multiple poromas will develop in a widespread distribution known as poromatosis

293. One of the following is not correct about Golimumab :

a. It is fully human recombinant IgG1 monoclonal antibody

b. It is approved for the treatment of psoriatic arthritis

c. It is contraindicated to be administred with methotrexate

d. It has specificity for human TNF -alpha

e. It is supplied in a 50 mg prefilled syringe or autoinjector that is administred s/c monthly

294. One of the following antiviral drugs does not have effect on CMV:

a. GANCICLOVIR

b. FOSCARNET

c. FAMCICLOVIR

d. CIDOFOVIR

e. ACYCLOVIR
295. Which of the following absorbable suture has poor ease of handling

a. Surgical gut

b. Polyglycolic acid

c. Polyglactin 910

d. Poliglecaprone 25

e. Glycomer 631

296. The least effect of UVB for photodamage is?

a. Sunburn

b. Photosensitivity

c. photoaging

d. SCC

e. Photoimmune suppression

297. Which of the following muscles is not innervated by the temporal branch of facial nerve:

a. Frontalis muscle

b. Corrugator supercilii Muscle

c. Orbicularis oculim (upper portion )

d. Orbicularis oculim (lower portion )

e. Auricular muscle ( anterior and superior portion)

298. Which of the following type of laser does not have water as target chromophor?

a. Diode

b. Holmium YAG

c. Erbium YAG

d. CO2
e. Pulsed dye

299.Which of the following is not correct about Lymphogranuloma venereum:

a. It caused by chlamydia trachomatis serevars L 1 – 3

b. The bubo is firm and painless

c. Urethra- genito- perianal syndrome

d. Pelvic inflammatory disease

e. Rectal strictures and stenosis

300. One of the following is inflammatory disorder that can flare immune reconstitution inflammatory
syndrome (IRS):

A. Eosinophilic folliculitis.

B. Dermatophytes.

C. Cytomegalovirus.

D. Varicella- zoster virus.

E. Demodex.

301. A 26 year old male presents with erythematous and scaly place bilaterally in groins area. Which of
the following diagnosis should not be considered in the differential diagnosis:

A. Erythrasrma.

B. Candidiasis.

C. Dermatophytes infection.

D. Lichen planus.

E. Psoriasis

302 . In parapsoriasis:

A. Large and small plaques present as infiltrated plaques.

B. The small plaque variant appear to exist in continuum with patch stage of mycosis fungoides.

C. Lesions in both variants are chronic and favor sun exposed skin.

D. Pathology consists of superficial mostly CD4+ infiltrate.


E. Best treatment is systemic corticosteroids.

303. The epidermis is a dynamic tissue in which cells are in constant motion. The kinetics of this motion
is characterized by:

A. Direction and speed are synchronous.

B. Keratinocytes pass each other but not melanocytes.

C. Movement is unidirectional.

D. Stability for direction and flow is provided by the basal membrane comples.

E. Lateral motion is slower than forward.

304. Itching in atopic dermatitis is characterized by:

A. Histamine is a key factor.

B. Partly produced by proteolytic enzymes.

C. Causes atrophy.

D. Its basis is unknown

E. Occurs mainly during the night.

305. DIF of DH reveals :

A. Linear band of IgG along the BMZ

B. Linear band of IgA along the BMZ

C. GranuLar deposits of IgG in the dermal papillae

D. GranuLar deposits of IgA in the dermal papillae

E. GranuLar and Linear band along BMZ

306. Which of the following is true about lichen sclerosis atrophicans:

A. Unknown to occur in children

B. Extragenital lesions are seen more frequently than genital lesions

C. The disease is benign

D. Antibodies to extracellular matrix proteins have been detected in the disease

E. Topical steroids are not helpful


307. The development of Lichen Plan is mediated through immune mecanisms. which of the following is
true regarding this?

A. The majority of lymphocytes in the infiltrate is CD4+

B. IgG is constantly found in the basement membrane.

C. Progression of the disease is led by CD4+

D. CD8+ lymphocytes with TCR gamma-beta is responsible for apoptosis

E. The basement membrane antigen involved is fibronectin

308. Propionbacterium acne is :

A. An aerobic organism

B. Located in hair follicule and does not diffuse through it

C. Activates TLR

D. Incapable of producing inflammatory mediators directly

E. Its density in hair follicule is inversely related to serum level of testosterone

309. which of the following is true about angioedema?

A. The allergic type is observed within 24-72 h after exposure to allergen

B. The pseudoallergic type is IgE mediated

C. ACE inhibitor induced type is mediated

D. Nonallergic type is histamin mediated

E. Acquired angioedema has increased function of C1- INH


310. A young man developed an asymptomatic slow growing skin colored nodule on his sole. The
histology showed sharply circumscribed tumor of cuboidal epittelial cells whith compact eosinpophilic
cytoplasma containing vacuoles. What is the diagnostic :

A. Nodular BCC

B. SCC

C. Hydradenoma

D. Poroma

E. Seborrheic keratosis

311. Thining , longitudinal ridges and distal splitting are nail changes that are typically found in :

a. Psoriasis

b. Onychomycosis

c. 20 nail dystrophy

d. Chronic eczema

e. Lichen Planus

312. Kasabach- Meritt syndrome is :

A. Infantile hemangioma

B. Rapidly involuting hemangioma

C. Kaposiform Hemangioendothelioma

D. Non-involuting congenital Hemangioma

E. Cherry angioma

313. A 38 Y old woman presented with loss of subcutaneous fat in The face and torso in a progressive
way. Which of these laboratory Tests would you like to check first :

A. CBC

B. Protein electrophoresis

C. Urine analysis

D. Serum lipids

E. Serum lipase
314. Which of the following is associated with Darrier disease?

A. Clear cell acanthoma

B. Acrokeratosis verruciform of Hopff

C. Syringocystadenoma papilliform

D. Fibrofolliculoma

E. Sebaceous adenoma

315. Cutaneous larva migrans is characterized by:

A. The infected human is a dead end host

B. Cutaneous lesions are mildly or non pruritic

C. The commonest site afflicted is the face

D. The folliculitis form lasts 6 – 8 months

E. The 1st line treatment is a 3 days course of albendazole

316. The eye is involved in leprosy in different ways. Which of these is the commonest:

A. Insensitive cornea

B. bleeding cornea

C. conjunctivitis

D. iritis

E. optic neuritis

317. Which of the following is not a histological feature of Polymorphic Light Eruption :

A. Epidermal spongiosis

B. Perivascular lymphocytic infiltrate

C. Scattered eosinophils and neutrophils

D. Normal papillary dermis

E. Sometimes it is difficult to distinguish from cutaneous Lupus Erythematous.


318. Woods lamp filter is made of:

A. Tin and chromium oxide

B. Nickel hydride and barium silicate

C. Nickel oxide and barium carbonate

D. Nickel oxide and barium silicate

E. Silicon dioxide

319. Which of the following is most commonly implicated in the production of AGEP ?

A. Beta blockers

B. Tetracyclines

C. NSAID

D. Lithium

E. Gold

320. Raindrops on a dusty road appearance is seen in a toxicity to:

A. Antimony

B. Beryllium

C. Arsenic

D. Bismuth

E. chromium

321. Which of the following is the most single characteristic finding in child abuse:

A. bruises

B. blunt trauma

C. pinch marks

D. buckles

E. loop marks
322. Which of the following is not true about pseudoxanthoma elasticum :

A. Autosomic Recessive disease

B. Mutation in ABCC6 gene

C. Present at birth

D. Lateral neck is usually affected first

E. Angioid streaks are suggestive of diagnostic

323. Which of the following is not associated with Elastosis Perforans Serpiginosa ?

A. Down syndrome

B. Ectodermal Dysplasia

C. Marfan syndrome

D. Osteogenesis imperfecta

E. Pseudoxanthoma elasticum

324 . Which of the following about hypertrophic scar is not true:

A. always preceded by injury

B. there is prominent erythema

C. does not contain myofibroblasts

D. no spontaneous healing

E. good response to treatment

325. Which of the following is a predominantly septal panniculitis ?

A. erythema induratum

B. pancreatic panniculitis

C. sclerema neonatorum

D. lipodystrophic panniculitis

E. alpha -1 antitrypsin defficiency


326. In which of the following antibiotic, the site of action is DNA gyrase :

A. vancomycin

B. clindamycin

C. sulfonamides

D. quinolones

E. trimethoprim

327. Which of the following antifungal is category D in pregnancy ?

A. Terbinafine

B. Voriconazole

C. Amphotericine B

D. Flucytosine

E. Caspofungin acetate

328. Collection of endothelial cells, pericytes and formation of vascular lumen, histologically favors the
diagnosis of:

A. Nevus flammus

B. Cystic hygroma

C. Hemangioma

D. Port-wine stain

E. lymphangioma

329. The most helpful investigative tool in diagnosis of lymphatic malformations is?

A. Ultrasonography

B. Biopsy

C. CT scan

D. Lymphography

E. Arteriography
330. IV IG is not used in :

A. Kawasaki dis

B. Erythrodermic pso

C. PV

D. Dermatomyositis

E. Necrotizing fasciitis

331. By dermatoscopy, melanin located at the papillary dermis appear as:

A. Black

B. White

C. Brown

D. Slate blue

E. Red - black

332. Regarding cryotherapy , One of the following is not true:

A. The freezing time is shorter for benign lesions than malignant

B. 10 sec. with open spray technique will cure 80% of AK

C. The freezing time is THE SAME for both cryoprob or spray technique

D. 2-3 mm lateral spread is sufficient for successful treatment of warts

E. Keloids can be treated with open spray technique alone or in combination with intralesional steroids.

333. Imiquimod is not used in :

A. lentigo maligna

B. AK

C. HSV

D. Exra-mammary Paget

E. keloids
334. Concerning treatment of malignancy with cryotherapy, One of the following is not true:

A. The entire tumor is generally frozen in one session

B. The recommended temperature is -50 to -60 at the base Of The tumor

C. It is not advised for treatment Of The tumor on the nose

D. Cure rate for BCC is over 90%

E. Freezing time is approximately 45 sec for a 1 cm lesion

335. Superantigen T – cell interaction lead to activation of :

A. 5 – 30% of entire circulatory T – cell population

B. O.o1% of entire circulatory T – cell population

C. 50% of entire circulatory T – cell population

D. 75% of entire circulatory T – cell population

E. Almost the entire circulatory T – cell population

336. One of the following is not increased in acne :

A. Substance P

B. 5- Lipooxygenase

C. Linoleic acid

D. Monounsaturated fatty acid

E. Corticotropin releasing hormone

337. The most common secondary proliferation in nevus sebaceous is :

A. Trichoblastoma

B. Syringocystadenoma papilliformis

C. Sebaceous adenoma

D. Trichilemmoma

E. BCC
338. Histologic profile of cantharidin blisters produced by blister beetle is similar to that seen in:

A. PV

B. BP

C. HAILEY- HAILEY Disease

D. Darier disease

E. Sub-corneal pustular drematosis

339. One of the following is more common in streptococcal rather than staphylococcal toxic shock
SYNDROME

A. Localized extremity pain

B. Diffuse macular erythema

C. Soft tissue infection

D. Mortality

E. Positive blood culture

all are more common in streptococcal except B

340.The mode of inheritance in which both parents are not affected is :

A. AR

B. AD

C. X- linked recessive

D. X – linked dominant

E. A semidominant
341. Infra patellar hypertrichosis is a cutaneous manifestation of:

A. Spinal dysraphism

B. Becker s nevus

C. Juvenile dermatomyositis

D. PCT

E. Coffin- siris syndrome

342. Streaks and whorls of hyperkeratosis following Blashko s lines describes:

A. Erythrokeratoderma variabilis

B. KID syndrome

C. CHILD syndrome

D. Chanarin – dorfman syndrome

E. Conradi- huerman – happle syndrome

343.. The least site to expect scabies mites on is the:

A. Wrists and hands.

B. Elbows.

C. Axilla.

D. Genitalia.

E. Buttocks.

344. The most typical presentation of microvasular occlusion syndrome:

A. Macular purpura.

B. Macular ecchymosis.

C. Livedo reticularis.

D. Inflammatory retiform purpura.

E. Non inflammatory retiform purpura

R
345. The most prevalent ophthalmologic finding in pseudoxanthoma elastic is:

A. Angioid streaks.

B. Optic drusen.

C. Owl s eye.

D. Macular degeneration.

E. Peau d orange color change of retinal epithelium.

346.The type of cutaneous T –cell lymphoma that has the worst prognosis is :bn

A. Primary cutaneous anaplastic large cell lymphoma

B. Sub-cutaneous panniculitis like T –cell lymphoma

C. Primary cutaneous CD4+small/medium pleomorphic T- cell lymphoma

D. Pagetoid reticulosis

E. Primary cutaneous gamma/ delta T- cell lymphoma

347. The earliest manifestation of chronic granulomatous disease is:

A. Oral ulcers resembling aphthous stomatitis.

B. Cutaneous granulomas.

C. Large poorly healing cutaneous ulcers.

D. Infection of the skin around ears and nose

E. Seborrheic dermatitis and scalp folliculitis.

348.The highest risk of mother to child transmission of untreated syphilis occur if the mother infected :

A. During the 1st seven months of pregnancy

B. 1 year before pregnancy

C. During the 8th or 9th month of pregnancy

D. 3-6 weeks before labor

E. 2 year before pregnancy


349.What is the prognosis of atopic eruption of pregnancy:

A. Flare after delivery

B. Premature delivery

C. Intrauterine fetal death

D. Good even in severe cases

E. Post- delivery haemorrhage

350. which is a neutrophilic dermatosis?

A. Lichen aureus

B. Granuloma faciale

C. Pyoderma gangrenosum

D. Erythema elevatum diutinum

E. Purpura annularis telangiectodes

351.In vogt – koyanagi – harada syndrome there is what :

A. Poliosis

B. Hypertrichosis

C. Café au lait macules

D. Palmar hyperkeratosis

E. Perioccular hyperpigmentation

352. Where androgen receptors are predominantly expressed:

A. Isthmus

B. Dermal papilla

C. Startumba# sale

D. Papillary dermis

E. Periglandulae dermis
353. One of the following is affected by loss- of – function mutations within the filaggrin gene:

A. Lamina lucida

B. Lamina densa

C. Hemidesmosome

D. Keratohyaline granule

E. Membrane coating granule

354.Glycine is a major component of a structure that is detected in Which of the following?

A. Cutis laxa

B. Dercum s disease

C. Asteatotic eczema

D. Penile fibromatosis

E. Ehler- danlos syndrome type I

355. Which of the following results from inherited anomalies in genes encoding connexins?

A. Diminished sweating

B. Nail- patella syndrome

C. NF-1

D. PPK

E. EBS ( Dwoling- Meara )

356. Spores resulting from fragmentation of hyphae into separate cells are What?

A. Microconidia

B. Macroconidia

C. Blastospores

D. Arthroconidia

E. Sporangiospores
357. Which disease is transmitted by affected males to their daugters but not to their sons?

A. Phenylketonuria

B. Incontinentia pigmenti

C. Acrodermatitis enteropathica

D. Anhidrotic ED

E. Hereditary haemorrhagic telangiectasia

358. The hallopeau type of pemphigus vegetans differs from Neumann type by what?

A. Vegetating erosions

B. More benign course

C. Intercellar IgG and C3

D. Involvment of oral mucosa

E. Antibodies 30 KDa PV antigen

359. In a female patient presenting with acne which of the following indicates to the testing for
hyperandrogenism ? :

A. Premenstrual flare

B. Submarine comedones

C. Pitted scars in the checks

D. Location of lesions in the trunk

E. Sudden onset of severe lesion

360. What causes thinning of the nail plate?

A. Old age

B. Acitretin

C. Tetracycline

D. Darrier s disease

E. Onychomatricoma
361. What is mixed type of panniculitis?

A. Neutrophilic panniculitis

B. Lupus profundus

C. Relapsing febrile nodular Panniculitis

D. Idiopathic nodular Panniculitis

E. Pancreatic panniculitis

362.In Which of the following pseudo- darrier sign is positive ?

A. Linear connective tissue naevus

B. Congenital smooth muscle hamartoma

C. Malignant peripheral nerve sheath tumor

D. Naevus lipomatodes cutaneous superficialis

E. Poroeratotic eccrine ostial and dermal duct naevus

363. UVB light is most effective in the treatment of pruritus associated with?

A. Hypothyroidism

B. Pharmacologic pruritus

C. Chronic renal failure

D. Diabetes mellitus

E. Lymphoma

364. What refers to anetoderma ?

A. Yellow thick wrinkled skin with elastotic degeneration

B. Cigarette paper – like scars and thin weakly polarized collagen

C. Circumscribed areas of slack skin with loss of elastis tissue

D. Streaks of atrophy. Telangiectasia and with adipose tissue beneath the epidermis

E. Streaks of atrophy. Telangiectasia and s/c fat immediately beneath the epidermis
365. Combining tetracycline and isotretinoine for the treatment of thr severe acne should be avoided
because of Which of the following ?

A. Multiple exuberant bone growth

B. Intracranial hypertension

C. Severe chilostomatitis

D. Nasal bleeding

E. Myalgia and arthralgia

366. Giemsa stain from a genital lesion shows shoal of fish (school of fish) arrangement. What is the
clinical presentation?

A. Pubic excoriations.

B. Painful ulcer.

C. Purulent urethral discharge.

D. Multiple itchy penile papules.

E. Crural erythematous scaly plaques with expanding borders.

367. A patient develops polyarteritis nodosa and orchitits. What is the likely etiology?

A. Hepatitis B.

B. HIV.

C. Mumps.

D. EBV.

E. HPV.

368.What refer to Cheiroarthropathy?

A. Pachydermodactyly

B. Diabetic thick skin

C. Restrictive dermopathy

D. Vinyl chloride induced scleroderma

E. Nephrogenic systemic sclerosis


369. Which of the following is associated with Piebaldism?

A. Myocardial infarction.

B. Blindness.

C. Hirschsprung s disease.

D. Basal cell carcinoma.

E. Multiple myeloma.

370.Patients with Darier s disease have an increased incidence of which of the following?

A. Kaposi s varicelliform eruption.

B. Kaposi s sarcoma.

C. Squamous cell carcinoma.

D. Basal cell carcinomas.

E. Malignant melanoma.

371.A 4-month-old infant has a skin-colored, non-tender, nodule on the bridge of the nose. The lesion
intermittently swells and becomes darker upon crying. What is the next most appropriate step?

A. Treatment with systemic steroids.

B. Biopsy.

C. Surgical excision.

D. Magnetic resonance imaging.

E. Observation.

372. In patients with Dermatomyositis, Anti-Jo1 is associated with what?

A. Renal involvement.

B. Pulmonary involvement.

C. Cardiac involvement.

D. Shawl sign.

E. None of the above.


373. A 17 year-old white male with past history significant for acne vulgaris presents with a 6 month
history of swelling of the forehead, nasal root, and eyelids. The symptoms have not responded to oral
antibiotics. The most likely diagnosis is:

A. Solid facial edema.

B. Erysipelas.

C. Myxedema.

D. B-Cell lymphoma.

E. Angiosarcoma.

374.Which of the following CHEMICAL peels requires either rinsing off with water or neutralization with
5% sodium bicarbonate after 2-4 mn to end its action ?

A. Glycolic acid

B. salicylic acid

C. trichloroacetic acid

D. lactic acid

E. jessner s solution

375. A patient presents with complaints of an urticaial- like rash periodically associated with lower
extremity bone pain and fever. The urticarial is not pruritic and you suspect a dg of Schnitzler synd.
Which of the following lab tests would support this dg?

A. Serum ferritin

B. Complement level

C. Serum and urine protein electrophoresis

D. RF

E. ANA
376. Which of the following cutaneous feature of relapsing polychondritis is indicative of an associated
myelodysplastic syndrome ?

A. Aphtous ulcers

B. Cutaneous small vessel vasculitis

C. Livedo reticulris

D. A + B

E. All of the above

377. Biopsy of suspected discoid lupus would show inflammation around Which portion of the hair
follicule ?

A. Hair bulb

B. Infundibulum

C. Isthmus

D. Bulge

E. Matrix

378.A patient presents with reccurent otitis media and atopic dermatitis , What synd may the child
have?

A. Chronic granulomatous dis

B. Wiskott-aldrich syndrome

C. Severe combined immunodeficiency

D. Job syndrome

E. Familial chronic muccocutaneous candidiasis

379. Microcystic adnexal carcinoma is most likely to occur on the?

A. Scalp

B. Neck

C. Back

D. Lips

E. Legs
380. What is the lag time between radiation exposure and development of non melanoma SCC ?:

A. 1 YEAR

B. 5 YEAR

C. 10 YEAR

D. 20 YEAR

E. 40 YEAR

381. A mutation in the gene on chromosome 17 that encodes the protein neurofibromin increases the
risk of Which malignancy ?

A. Prolactinoma

B. Mucosal neuroma

C. Acute lymphocytic leukemia

D. Juvenile myelomonocytic leukemia

E. Progressive multifocal leukoencephalopathy

382. A patient presents with complaints of dry eyes and mouth giving her mucosa a gritty feeling. She
has a family history of SLE . Which of the following AUTOANTIBODIES is most specific for this diagnosis ?

A. SS-A

B. SS-B

C. Alpha-fodrin

D. dsDNA

E. None of the above

383.Which of the following diagnosis tools cannot be used to follow and monitor muscle inflammation
in Dermatomyositis?

A. EMG

B. CT- SCAN

C. MRI

D. ULTRASOUND

E. Serum level of muscle enzymes


384. The most prevalent ophthalmologic finding in pseudoxanthoma elastic is:

A. Angioid streaks.

B. Optic drusen.

C. Owl s eye.

D. Macular degeneration.

E. Peau d orange color change of retinal epithelium.

385. . High ratio of maternal urinary estrogen precursors in a male fetus suggests the diagnosis of ?

A. Refsum s dis

B. Sjogren-larsson synd

C. X – linked ichtyosis

D. CHILD Syndrome

E. Neutral lipid storage disease

386. Which of the following is NOT correct about clinical features of immune deficiency caused by HIV -
2:

A. 5 – 8 folds less transmissibility

B. More vertical transmission

C. Longer period of latency

D. Slower rate of CD4+ cell count decline and clinical progression

E. The outcome among HIV - 2 infected patients may be slightly better

387. Which of the following is affected by loss – of function mutations within fillagrin gene?

A. Lamina lucida

B. Lamina densa

C. Hemidesmosome

D. Keratohyaline granule

E. Membrane coating granule


388. What is The 2 most common neoplasm in Gorlin syndrome:

A. Medulloblastoma

B. Meningioma

C. Melanoma

D. Renal cell carcinoma

E. SCC

389. Which of the following is NOT CORRECT about special stains in dermatology:

A. Crystal violet stains amyloid metachromatically purple with blue background

B. Fite – faraco Stains mycobacterium leprae blue

C. Orcien Stains muscle and nerve yellow

D. Sudan black Stains lipids black

E. Colloidal iron stains acid mucopolysaccharides blue

390. Which of the following toll-like receptor is involved in recognizing lipopolysaccharide?

A. TLR2.

B. TLR3.

C. TLR4.

D. TLR5.

E. TLR7.

391. Which of the following is needed by tyrosinase to function?

A. Zinc.

B. Iron.

C. Selenium.

D. Magnesium.

E. Copper
392.The cellular source of antibacterial peptide Dermcidin is:

A. Keratinocytes.

B. Airway epithelia.

C. Granulocytes.

D. Intestinal tract.

E. Sweat glands

393. The most frequently observed dermatologic manifestations of acute hepatitis C is:

A. Lichen planus.

B. Polyarteritis nodosa.

C. Leukocytoclastic vasculitis.

D. Necrolytic acral erythema.

E. Porphyria cutanea tarda

394.. Movement of basal cells in basal layer is facilitated by:

A. Crowding of cells resulting from mitosis.

B. Made possible by degradation of anchoring filaments.

C. Assoicated with downgrading of integrin alpha 6 beta 4.

D. Mediated through cytokines arriving from basement membrane.

E. Controlled by growth factors.

395.One of the followings is not correct about benign cephalic histocytosis:

A. Infants < 1 year most commonly affected

B. Red to brown macules and papules on face and neck

C. Treatment is required

D. Ultrastructural finding : worm – like bodies

E. Histiocytes express CD 11 , CD 14 . CD 68
396. One of the following drugs is not a cause of small vessel vasculitis

A. Opiates

B. Barbiturates

C. Amphetamines

D. Cocaine

E. Marijuana

397. One of the following photosensitivity disorder has action spectrum 290 – 340 nm :

A. Cocaine syndrome

B. XP

C. UV- sensitive syndrome

D. Trichothiodystrophy

E. Bloom syndrome

398.HIV infection may be associated with One of the following granulomatous dermatitis :

A. Necrobiosis lipoidica

B. Classic granuloma annulare

C. Annular elastolytic giant cell granuloma

D. Cutaneous crohn s disease

E. Sarcoidosis

399.Necrotic material is the perforating substance of One of the following major diseases:

A. Reactive perforating collagenosis

B. Elastosis perforans serpiginosa

C. Acquired perforating dermatosis

D. perforating periumbilical calcific elastosis

E. perforating folliculitis
400. Histopathological reaction of the type of tuberculoid granuloma with caseation may be csused by
One of the following foreign substances :

A. Berylium

B. Paraffin

C. Talc

D. Tatoo ink

E. Silicone

401. one of the following IS NOT minor WHO criteria for diagnosis of systemic mastocytosis:

A. More than 25% of mast cells in bone marrow samples or extracutaneous lesions are spindle shaped or
atypical.

B. Extracutaneous mast cells (CD117+) EXPRESS CD2, CD25

C. Presence of C- kit condon on 814 mutation in blood, bone marrow or extracutaneous tissues.

D. Serum total tryptase level is persistently > 20 mg/ ml

E. Multifocal dense infiltates of mast cells in bone marrow or extracutaneous tissues

402. . Pharmacologic pruritus of idiopathic pathomechanism is seen with:

A. Gold salts.

B. Beta- blockers.

C. 8- methoxypsoralen.

D. Tramadol.

E. Chloropromazine.
403. One of the following is NOT a component of the epidermal differentiation complex:

A. Loricrin.

B. Involucrin.

C. S-100 calcium-binding proteins.

D. Small proline-rich peptides.

E. Transglutaminase 1

404. Which of the following drugs causes cholestasis:

A. Morphine.

B. Retinoids.

C. Erythromycin estolate.

D. Chlorquine.

E. Isoniazide.

405. Early cornification can be observed within the hair canal at approximately:

A. 12 weeks estimated gestational age (EGA).

B. 18 weeks EGA.

C. 15 weeks EGA.

D. 22 weeks EGA.

E. 8 weeks EGA.

406. Staphylococcal scalded skin syndrome (SSSS) is associated with which of the following?

A. Desmocollin 1.

B. Desmocollin 3.

C. Desmoglein 1.

D. Desmoglein 3.

E. None of the above.


407.Which one of the following is NOT a predisposing factor to colonization and infection with MRSA?

A. Age (older than 65)

B. Prior antibiotic therapy

C. Recent hospitalization

D. Vaccination

E. Chronic illness.

408. The infraorbital foramen is located:

A. 5 cm below the infraorbital rim in the midpupillary line

B. 1 cm below the infraorbital rim in the midpupillary line

C. 2 cm below the infraorbital rim in the midpupillary line

D. 3 cm below the infraorbital rim in the midpupillary line

E. 4 cm below the infraorbital rim in the midpupillary line

409. Which one of the following is not an important risk factor for arterial Ulcers?

A. Cigarette smoking

B. Age >40 years

C. Hyperlipidemia

D. Female

E. Hyperhomocysteinemia

410. Which one of the following is true regarding Junction lines?

A. Are skin tension line

B. Refer to the normal skin-graft interface of a grafted defect

C. Are difficult to ascertain in most patients

D. Separated cosmetic units

E. Not useful for tissue matching


411. In which one of the following diseases the activity of 2,3-dioxygenase is increased?

A. Involuting phase of hemangiomas

B. Proliferating phase of hemangiomas

C. Normal vascular development

D. Normal lymphatic development

E. Resting phase of hemangiomas

412. One of the followings does not trigger the development and / or growth of melanocytes nevi:

A. PREGNANCY

B. Growth hormone

C. Parathyroid hormone

D. Thyroid hormone

E. Adissson disease

413. One of the followings is not predisposing for the development of SSC :

A. Chronic non healing wounds

B. Long standing DLE

C. Erosive LP

D. PSO

E. Linear porokerarosis

414. One of the followings types of BCC has high recurrence rate :

A. Micronodular BCC

B. Fibroepithelioma of Pinkus

C. Nodular BCC

D. Superficiel BCC

E. Morpheaform BCC
415. One of the following tumors has location in mouth as a major distinguishing feature from other
tumors :

A. Adenoid cystic carcinoma

B. Ameloblastoma

C. Merkel cell carcinoma

D. Sebaceous carcinoma

E. Trichoepithelioma

416.Acquired cutis laxa may be associated with One of the following drugs :

A. PENICILLIN

B. DOXYCYCLIN

C. DAPSONE

D. METHOTREXATE

E. CYCLOSPORINE

417. One of the followings is not correct about Actinic Prurigo:

A. Childhood onset may persist for years

B. Papules and nodules with hemorrhagic crusts in sun exposed sites

C. Cheilitis and conjunctivitis are one of the clinical features

D. Strong association with HLA- DR8

E. MANAGEMENT includes photoprotection , NB – UVB, PUVA , Thalidomide.

418.One of the followings is not an acute clinical feature of arsenic poisoning:

A. Loss of hair and nails

B. Remaining nails show Mee s lines after 8 weeks

C. Respiratory failure

D. Alopecia

E. Acrodynia
419.One of the followings is not correct about Pseudoxanthoma elasticum :

A. An AR disorder

B. Cutaneous feature include yellowish skin papules, cobblestoning and redundant folds in flexural sites.

C. Angeoid streaks

D. Clumped and distorted elastic fibers with deposites of Ca

E. Calcification of elastic fibers in small – sized arteries

420. HIV infection may be associated with One of the following granulomatous dermatitis :

A. Necrobiosis lipoidica

B. Classic granuloma annulare

C. Annular elastolytic giant cell granuloma

D. Cutaneous crohn s disease

E. Sarcoidosis

421.One of the following IS NOT CORRECT about scleroderma adultorum of Buscke in Diabetes Meletus :

A. Occurs in 30% of DM

B. The condition is malignant

C. Common condition

D. Occurs in children

E. Lesions have broad area of non - pitting oedema

422. One of the following IS NOT CORRECT about early indeterminate leprosy:

A. The first clinical feature is hyperpigmented nodule

B. The first clinical sign is numbness

C. The first Lesion is hypopigmented macule

D. Peripheral nerves not enlarged

E. Plaques and nodules do not occur


423. One of the following IS NOT CORRECT about Pancreatic Panniculitis:

A. There are subcutaneous nodules without other symptoms

B. Associated with acute and chronic Pancreatitis and Pancreatic carcinoma

C. Lobular or mixed Panniculitis

D. Deposition of basophilic material due to saponification of fat by calcium salts

E. Treatment of underlying Pancreatic disorders

424. One of the followings IS NOT CORRECT regarding penciclover:

A. Mechanism of action similar to acyclovir

B. It reaches higher intracellular concentration than acyclovir

C. It is less potent than acyclovir because it does not cause chain termination of viral DNA

D. It has a poor bioavailability than acyclovir

E. It is available in tablets and topical forms

425. One of the followings IS NOT CORRECT about pyogenic granuloma:

A. Rapidly growing red papule or polyp of skin or mucosa.

B. May be arising on the gingiva of pregnant women.

C. Consist of lobules of small capillaries set in a fibromyxoid matrix.

D. There is a granulomatous reaction in histology.

E. Pulsed dye laser has been to be a safe and effective treatment for small pyogenic granuloma.

426. One of the followings types of cutaneous cysts IS NOT composted of stratified squamous
epithelium:

A. Epidermoid cyst.

B. Dermoid cyst.

C. Hidrocystoma cyst.

D. Vellus hair cyst.

E. Steatocystoma.
427. One of the followings IS NOT a hupercoagulability intravascular cause of acquired livedo reticularis:

A. Antiphospholipid syndrome.

B. Protein S and C deficiencies.

C. Factor V Leiden mutation.

D. Homocystinuria.

E. Cutaneous polyarteritis nodosa.

428. One of the following IS NOT CORRECT about Greither syndrome:

A. Diffuse palmoplantar keratoderma which is transgredient

B. Hyperhidrosis

C. Histologically epidermlytic changes of the granular layer are seen

D. Protein production of mutated gene is K9( k1 is correct)

E. It is AD inheritance

429. One of the following disease IS NOT a secondary disease association with cutis verticis gyrata:

A. Acromegaly

B. Klinefelter syndrome

C. Cataract

D. Tuberous sclerosis

E. Noonan syndrome

430. One of the following IS NOT CORRECT regarding dermatological manifestations of congenital
generalized lipodystrophy:

A. Acanthosis nigricans, often early onset and wide spread.

B. Hypertrichosis, including increased scalp hair at birth

C. Hyperhidrosis

D. Ichtyosis vulgaris

E. Xanthomas

Вам также может понравиться